Download as docx, pdf, or txt
Download as docx, pdf, or txt
You are on page 1of 71

SANS-TRAUMA

1. A LP in a pt with pseudotumor cerebri will show which of the following?

-Elevated opening pressure


-Pleocytosis
-Elevated IgG index
-Low glucose
-High protein

Elevatedopeningpressure.
Pseudotumor cerebri is a disorder of idiopathic intracranial hypertension (IIH). It often affects
young, obese women who p/w H/A, papilledema, and elevated LP opening pressure. The
chemical and cellular composition of the CSF is usually normal. 1st line Rx is typically
administration of acetazolamide wwo furosemide. If vision is acutely threatened, temporary CSF
drainage by LP or LD may allow a trial of these diuretics. Options for more permanent treatment
are optic nerve sheath (ONS) fenestration and VPS. Although the latter is the gold standard, it's
often fraught w complications over time in this pt population. The venous sinuses and jugular
veins should be imaged as stenting can be considered for stenosis as an alternative to VPS.

2. 67M who receives HD has acute onset large SDH. Surgical evacuation is complicated by
difficulty controlling bleeding from the scalp and the contused brain surface. Hemoglobin is 8,
plt 162K, prothrombin time (PT) is 13.2s (N 11-13.5). The patient is HDS. Which of the
following is the most appropriate next step in mgt?

-Administer DDAVP
-Administer FFP
-Administer TXA
-Administer 4-factor PCC
-Administer protamine
DDAVP.
Discussion:
Uremia/renal failure pts develop an acquired deficiency in plt function caused by decreased
thromboxane A2 function, increased platelet-inhibitory prostaglandin, and excessive nitric
oxide synthesis. In pt who is HDS w/difficult to control intraop bleeding, administration of
cryoprecipitate or DDAVP would be the best initial step in mgt. Postop dialysis may be
considered as well. DDAVP increases the plasma levels of factor VIII and vWF and shortens
the partial thromboplastin time (PTT) and bleeding time. DDAVP has no effect on plt count
or aggregation, but it enhances plt adhesion to the vessel wall. A short-lived effect of DDAVP
is the release of large amt of tPA into the plasma.
References:
Escolar G, Díaz-Ricart M, Cases A. Uremic platelet dysfunction: past and present. Curr Hematol
Rep. 2005 Sep;4(5):359-67.Medow JE, Dierks MR, Williams E, et al. The Emergent Reversal of
Coagulopathies Encountered in Neurosurgery and Neurology: A Technical Note. Clin Med Res.
2015; 13(1): 20-31.Hedges SJ, Dehoney SB, Hooper JS, Amanzadeh J, Busti AJ. Evidence-based
treatment recommendations for uremic bleeding. Nat Clin Pract Nephrol. 2007 Mar;3(3):138-53.
doi: 10.1038/ncpneph0421. PMID: 17322926.Frontera JA, Lewin JJ 3rd, Rabinstein AA, Aisiku
IP, Alexandrov AW, Cook AM, del Zoppo GJ, Kumar MA, Peerschke EI, Stiefel MF,
Teitelbaum JS, Wartenberg KE, Zerfoss CL. Guideline for Reversal of Antithrombotics in
Intracranial Hemorrhage: A Statement for Healthcare Professionals from the Neurocritical Care
Society and Society of Critical Care Medicine. Neurocrit Care. 2016 Feb;24(1):6-46. doi:
10.1007/s12028-015-0222-x. PMID: 26714677.
Read More
3. Which of the following tests is most appropriate for differentiating b/w a traumatic LP and a
pre-existing SAH?

-CSF protein
-CSF color
-CSF cell count
-CSF clarity
-CSF glucose
CSF color. Xanthochromia refers to the yellow-orange discoloration of CSF, most often caused
by lysis of RBCs. Discoloration begins after RBCs have been in spinal fluid for about 2h. While
it sometimes can be seen visually, detection of xanthrochromia via spectrophotometry is the most
reliable method of determining whether a SAH has occurred.

4. Which of the following is the current "gold standard" for ICP monitoring & mgt?

-There is no gold standard for ICP monitoring.


-LP
-TCD
-Fiberoptic intraparenchymal monitor
-EVD w/fluid coupled pressure transducer
EVD.
The gold standard for ICP monitoring and mgt is the EVD with a fluid coupled pressure
transducer. Fiberoptic monitors can also monitor ICP accurately, but do not allow for removal of
spinal fluid. Fiberoptic monitors historically suffer from measurement drift and cannot be
recalibrated unless replaced. LP allows for CSF drainage & pressure measurements in pts w/
communicating hydrocephalus. It is not accurate in patients with non-communicating
hydrocephalus.

5. An 80M suffers L aSDH s/p craniotomy. Following surgery, he regains a withdrawal response
to painful stimuli in the R arm + leg. 6h later, he is found to have new right gaze deviation and
has lost this R extremity motor response. CTH shows no blood reaccumulation or shift. Which of
the following is the most appropriate course of action?

-RTOR
-Administer IV lorazepam
-MRI with DWI
-CTA & CTP
-Stat EEG
Administer IV lorazepam.
Although a forced gaze & hemiparesis can be caused by stroke, the gaze preference of a stroke is
typically opposite the side of paralysis. It would therefore be unlikely that a CTA/P or MRI
would add addt'l actionable info in the short term. The pt has already had a CT to r/o post-
operative hemorrhage. The most likely diagnosis is non-convulsive status. The onset of the
seizure is unknown. A stat EEG (3) might be useful, but there is enough info to start Rx with a
benzo such as lorazepam (4mg IV one time, repeated x1 if sz persist). A 2nd longer acting agent
such as fosphenytoin, valproate, or levetiracetam should be loaded as soon as feasible. At times a
SDD laying on the surface of the brain may be implicated as a source of seizure but RTOR to
remove a drain would probably only be done in medically refractory cases.

6. (Bank #1) Severe hypomagnesemia is most likely to produce which of the following?

-Hyperkalemia
-Hypotension
-Ventricular arrhythmias
-Hypoactive reflexes
-Hypercalcemia
Ventricular arrhythmias.
Discussion:
Mg prevents increases in action potential duration and prolongation of membrane repolarization.
These changes commonly occur after myocardial ischemia and can lead to ventricular
arrhythmias. A-Fib can also be provoked by hypoMg. Magnesium causes presynaptic inhibition
leading to a depressant effect on the CNS. HypoMg results in renal potassium loss and also
suppresses PTH hormone release and activity. HypoMg therefore often occurs in conjunction
w/hypokalemia & hypocalcemia. HypoMg can be associated w/HTN rather than HoTN.
References:
1. Agus MS, Agus ZS. Cardiovascular actions of magnesium. Crit Care Clin. 2001
Jan;17(1):175-86. 2. Handb Clin Neurol. 2017;141:705-713.

7. Acute respiratory distress syndrome (ARDS) is the result of

-Atelectasis
-Pulmonary fibrosis
-Diffuse alveolar hemorrhage
-Pneumonia
-Congestive heart disease
Pneumonia.
Discussion:
ARDS can result from direct or indirect lung injury. Pneumonia (bacterial, viral, fungal, or
opportunistic) is the most common DIRECT lung-injury leading to ARDS. Aspiration of
gastric contents, pulmonary contusions, inhalation injury, and near drowning can also lead to
ARDS. Sepsis is the most common cause of INDIRECT lung-injury leading to ARDS.
ARDS can also be caused by trauma or hemorrhagic shock, pancreatitis, burn injury, and blood
product transfusion. After direct or indirect lung injury, lung alveolar macrophages are activated,
leading to the release of potent proinflammatory mediators and chemokines promoting the
accumulation of neutrophils and monocytes. This exudative phase of ARDS leads to alveolar and
microvascular damage and loss of barrier function followed by alveolar flooding. Tumor
necrosis factor (TNF) mediated expression of tissue factor promotes platelet aggregation and
microthrombus formation, as well as intra-alveolar coagulation and hyaline membrane
formation. The exudative phase is followed by a proliferative phase. After epithelial integrity has
been reestablished, the reabsorption of alveolar edema in the provisional matrix restores alveolar
architecture and function. A final fibrotic phase has been linked to prolonged mechanical
ventilation and increased mortality.
References:
1. N Engl J Med. 2017 Aug 10;377(6):562-572. 2. Crit Care. 2018 Oct 26;22(1):280.

8. The mean arterial blood pressure (MAP) at normal resting heart rate is best approximated by
which of the following formulas, where SBP is the systolic pressure and DBP the diastolic
pressure?

-MAP = DBP + 1/4(SBP-DBP)


-MAP = SBP - 1/2(DBP)
-MAP = DBP + 1/3(SBP-DBP)
-MAP = DBP + 1/2(SBP-DBP)
-MAP = SBP - 1/3(DBP)
MAP = DBP + 1/3(SBP-DBP).
MAP is average arterial pressure throughout 1 cardiac cycle (systole and diastole). MAP is
influenced by CO and SVR. Most commonly used formula to estimate MAP is: MAP = DBP +
1/3(SBP-DBP), where SBP = systolic blood pressure and DBP = diastolic blood pressure.

9. (Bank #1) Which of the following factors or findings would warrant a non-contrast CTH 2h
after minor closed head injury?

-GCS < 15
-Vomiting, 2 or more episodes
-Suspected open, depressed, and/or basal skull fx
-Age 65+
-Any of the above
Any of the above.
Discussion:
Significant variations exist in the utilization of CT for detection of traumatic intracranial
pathology among patients with minor head injuries. The Canadian CT Head Rule was derived
from a prospective cohort of Canadian emergency department patients with a GCS of 13-15 after
head injury. Five high-risk factors and two additional medium-risk factors were identified. The
presence of any one of them indicates need for a head CT. The high-risk factors were 100%
sensitive for predicting need for neurosurgical intervention, and would require only 32% of
patients to undergo CT. The rule is not applicable if the patient did not experience a trauma, has
a Glasgow Coma Scale score lower than 13, is younger than 16 years, is taking warfarin or has a
bleeding disorder, or has an obvious open skull fracture.
High Risk for Neurosurgical Intervention
-1. GCS < 15 at 2h after injury
-2. Suspected open or depressed skull fracture
-3. Any sign of basal skull fracture *
-4. Two or more episodes of vomiting
-5. 65 years or older
Medium Risk for Brain Injury Detection by Computed Tomographic Imaging
-6. Amnesia before impact of 30 or more minutes
-7. Dangerous mechanism **
* Signs of basal skull fracture include hemotympanum, racoon eyes, cerebrospinal fluid, otorrhea
or rhinorrhea, Battle’s sign.
** Dangerous mechanism is a pedestrian struck by a motor vehicle, an occupant ejected from a
motor vehicle, or a fall from an elevation of 3 or more feet or 5 stairs.
References:
Stiell IG, Wells GA, Vandemheen K, et al. Variation in ED use of computed tomography for
patients with minor head injury. Ann Emerg Med. 1997; 30:14-22.Stiell IG, Wells GA,
Vandemheen K, et al. The Canadian CT Head Rule for patients with minor head injury. Lancet.
2001; 357:1391-6.

10. What are the (5) 'high risk factors for neurosurgical intervention' to get CTH following CHI?
1. Glasgow Coma Scale score lower than 15 at 2 hours after injury
2. Suspected open or depressed skull fracture
3. Any sign of basal skull fracture *
4. Two or more episodes of vomiting
5. 65 years or older

* Signs of basal skull fracture include hemotympanum, racoon eyes, cerebrospinal fluid, otorrhea
or rhinorrhea, Battle's sign.

11. What are the (2) 'medium risk factors for neurosurgical intervention' to get CTH following
CHI?
6. Amnesia before impact of 30 or more minutes7. Dangerous mechanism **

** Dangerous mechanism is a pedestrian struck by a motor vehicle, an occupant ejected from a


motor vehicle, or a fall from an elevation of 3 or more feet or 5 stairs.

12. 26M p/w severe H.A and new-onset sz. MRI Brain with SSS thrombosis and small R parietal
hemorrhagic venous infarction causing minimal mass effect. Patient is lethargic but easily
arousable, follows complex commands on the right. Which of the following is the most
appropriate initial mgt strategy?

-Ventriculostomy
-Transvenous endovascular thrombectomy
-Craniotomy and evacuation of the hematoma
-IV anticoagulation
-Mannitol and serial imaging
IV anticoagulation. Pt has had hemorrhagic venous infarction d/t thrombosis of SSS. Sz are
common presenting symptom of ICH. Current AHA/ASA guidelines suggest IV anticoagulation
as initial mgt strategy of venous sinus thrombosis, even in presence of ICH. Mannitol is probably
not indicated in this pt as there is little mass effect and dehydration may precipitate further
thrombosis. This patient is not suffering from significant ME, so clot evacuation is not indicated
and would delay administration of anticoagulation. The pt. is arousable and doesn't ahve HCP,
therefore CSF drainage and invasive ICP monitoring via a ventriculostomy are not indicated.
Transvenous endovascular thrombectomy can be considered in patients who are refractory to
medical mgt or perhaps to patients who present in extremis, but is not typically 1st line therapy.

13. Which of the following conditions increases the cerebral metabolic rate (CMRO2)?

-Barbiturate coma
-Hyperoxygenation
-Brain injury
-Fever
-General anesthesia
Fever. Rate of oxygen consumption by the brain is known as cerebral metabolic rate of oxygen
(CMRO2). The rate of oxygen consumption is decreased by sedatives such as barbiturates and
GA. Brain injury will aso tend to reduce brain metabolism, particularly in the acute phase.
Hyperoxia on its own will not stimulate brain metabolism. Fever has been shown to increase
CMRO2. This is a major reason why prevention of fever is considered important in patients who
are prone to brain ischemia.

14. Which of the following are the most common findings of CSW syndrome?

-Hypernatremia, elevated urine Na, elevated UOsm, hypovolemia


-Hyponatremia, elevated urine Na, decreased UOsm, hypovolemia
-Hypernatremia, elevated urine Na, elevated UOsm, hypervolemia
-Hyponatremia, elevated urine Na, elevated UOsm, hypovolemia
-Hyponatremia, decreased urine Na, elevated UOsm, hypovolemia
HypoNa, Elevated UNa, Elevated UOsm, hypovolemia. Evaluation for CSW begins with a BMP
to identify the hyponatremia. Urine studies are commonly checked for urine sodium and
osmolality. Urine sodium is typically elevated above 40 meq/L. Urine osmolality is elevated
above 100 mosmol/kg. The patient must also have signs or symptoms of hypovolemia such as
hypotension, decreased central venous pressure, lack of skin turgor, or elevated hematocrit.
SIADH will have a similar laboratory picture as cerebral salt wasting with hyponatremia and
increased urine sodium. However, with SIADH, the patient is euvolemic to hypervolemic from
the retained free water as compared to the hypovolemic picture of cerebral salt wasting.

15. Several days s/p endovascular coiling of a rupture aneurysm with Fisher grade 3 SAH, a
patient has hypoNa. Which of the following findings would support dx of CSW rather than
SIADH?

-Hypovolemic state
-Increase [UNa]
-Low serum urate levels
-Low urine output
-High UOsm
Hypovolemia is key distinguishing factor in discerning b/w CSW and SIADH secretion. Low
urine output is seen in SIADH but not CSW. High UOsm, increase [UNa], and low serum urate
levels are seen in both CSW and SIADH.

16. Pseudohyponatremia is most likely in patients with which of the following conditions?

-Hypertriglyceridemia
-Diabetes insipidus
-Carbamazepine use
-SSRI use
-Septic shock
Hypertriglyceridemia.
Discussion:
PseudohypoNa consists of a state where there is a reduction in the % of water in plasma relative
to sodium, such as a state of hypertriglyceridemia, where there is a lower sodium concentration
as a result of high triglycerides that reduces the percentage of water, and thus falsely lowering
the concentration of sodium. SSRI and carbamazepine result in hyponatremia due to SIADH and
excessive water retention due to elevated ADH secretion. Diabetes insipidus results in
hypernatremia.
References:
Wang Y, Attar BM, Abu Omar Y, Agrawal R, Demetria MV. Pseudohyponatremia in
Hypertriglyceridemia-Induced Acute Pancreatitis: A Tool for Diagnosis Rather Than Merely a
Laboratory Error? Pancreas. 2019 Jan;48(1):126-130; Adashek ML, Clark BW, Sperati CJ,
Massey CJ. The Hyperlipidemia Effect: Pseudohyponatremia in Pancreatic Cancer. Am J Med.
2017 Dec;130(12):1372-1375.

17. Inappropriate secretion of ADH can be distinguished from fluid volume overload most
accurately by which of the following?

-Clinical examination
-UNa
-UNa
-SOsm
-UOsm
Clinical examination. Fluid overload can best be distinguished from SIADH by clinical exam.
Dx of SIADH requires an examination c/w euvolemia. Volume overload can be caused by
several conditions such as heart failure, liver failure, nephrotic syndrome, or renal failure.
Clinical features c/w volume overload include jugular venous distention, peripheral edema,
pulmonary edema, and ascites. SOsm will be low in both SIADH & hypervolemia. UNa is
typically >20 mEq/L and urine osmolality > 10 mOsm in SIADH. While urine sodium is < 20
mEq/L in heart failure, ascites, and nephrotic syndrome, hypernatremia due to renal failure may
present with urine sodium well in excess of 20 mEq/L.
18. Which of the following is the normal global cerebral blood flow (CBF)?

-50 mL/ (100g/min)


-40 mL/ (100g/min)
-20 mL/ (100g/min)
-70 mL/ (100g/min)
-60 mL/ (100g/min)
50. CBF = volume of blood that flows per unit mass per unit time in brain tissue. The normal
value of CBF in adults is about 50 mL/100g/min. Lower values can occur in white matter
[approx. 20mL/(100g/min)] and greater values can occur in gray matter [approx. 80 mL/
(100g/min)]

19. 25M brought to ED for L parasternal stab wound. Blood pressure is 70/50 mmHg, neck veins
are distended, and heart sounds are muffled. Which of the following is the most likely diagnosis?

-Pericardial tamponade
-Disruption of sympathetic fibers
-Vasovagal syncope
-Pneumothorax
-Acute blood loss anemia
Pericardial tamponade. The patient has shock related to this stab wound. Beck's triad of HoTN,
elevated systemic venous pressure (neck vein distension), and muffled heart sounds is present &
indicated of pericardial tamponade (a form of obstructive shock). Pneumothorax is possible, but
decreased breath sounds would be a dominant feature. Hemorrhagic shock would not have
distended neck veins, and shock related to SCI would not have these features.

20. When treating a patient with acute symptomatic hypoNa with hypertonic saline, the
maximum correction of plasma sodium levels in the first 24h is

- >15 mEq/L
- 1-4 mEq/L
- 12-15 mEq/L
- 4-8 mEq/L
- No current guidelines
4-8. Sodium correction is based on the acuity of hyponatremia and the severity of symptoms. In
acute symptomatic hyponatremia, a correction of 4-6 mEq/L (up to 8mEq/L) is safe and can be
achieved within a few hours, with maintenance of stable sodium levels for up to 24 hours. A
more rapid correction can lead to osmotic demyelination syndrome with neurologic sequelae
thereof.

21. 64F evaluated for painful, swollen, warm R calf 6 wks s/p lumbar laminectomy. Doppler US
shows extensive DVT. Which of the following is the most appropriate next step in mgt?

-Placement IVCf
-Initiation of oral anticoagulation therapy
-Initiation of IV anticoagulant therapy
-No treatment is needed
-Initiation of antiplatelet medication
Oral anticoagulation. For primary treatment of patients with DVT and/or PE, whether provoked
by a transient risk factor or by a chronic risk factor or unprovoked, the ASH guideline panel
suggests using a shorter course of anticoagulation for primary treatment (3-6 months) over a
longer course of anticoagulation for primary treatment (6-12 months). Oral anticoagulation is the
preferred method. These recommendations apply to patients who are eligible to receive
anticoagulation. For patients with a contraindication to anticoagulation, insertion of a retrievable
IVC filter may be indicated, with retrieval as soon as the patient is able to receive
anticoagulation.
22. 67 yo pt is hospitalized for mgt of an acute ischemic stroke. In the ED, the patient receives
IV tPA and then sustains an IPH. Which of the following is the most appropriate pharmacologic
reversal agent for this pt?

-Protamine
-Cryoprecipitate
-FFP
-Vitamin K
-4-factor prothrombin complex concentrate (PCC)
Cryoprecipitate.
Discussion:
The reversal agent of choice for tPA associated hemorrhages is cryoprecipitate which contains
fibrinogen, which is depleted by tPA. If cryoprecipitate is unavailable, fresh frozen plasma (FFP)
and tranexamic acid can be considered. There is no role for vitamin K or 4-factor PCC.
References:
Frontera JA, et al. Guideline for reversal of anti-thrombotics in intracranial hemorrhage: a
statement for healthcare professionals from the neurocritical care society and society of critical
care medicine. Neurocrit Care. 2016 Feb;24(1):6-46; Saghi S, Willey JZ, Cucchiara B, Goldstein
JN, Gonzales NR, Khatri P, Kim LJ, Mayer SA, Sheth KN, Schwamm LH; American Heart
Association Stroke Council; Council on Cardiovascular and Stroke Nursing; Council on Clinical
Cardiology; and Council on Quality of Care and Outcomes Research. Treatment and Outcome of
Hemorrhagic Transformation After Intravenous Alteplase in Acute Ischemic Stroke: A Scientific
Statement for Healthcare Professionals From the American Heart Association/American Stroke
Association. Stroke. 2017 Dec;48(12):e343-e361.

23. 36M has closed fx of femur and small, focal area of SAH after being involved in MVC. No
other cranial injury is noted, and neurologic exam is normal. After repair of the femur fx, the
patient does not arouse from anesthesia. CTH shows bilateral diffuse, small, hypodense lesions.
Which of the following is most likely cause of the change in this patient's clinical status?

-Watershed infarcts from hypotension


-Posterior reversible encephalopathy syndrome (PRES)
-Diffuse embolic infarcts
-Diffuse axonal injury (DAI)
-Fat emboli
Fat emboli. The presence of femur fx and diffuse hypodense lesions would be indicative of fat
emboli. One would expect DAI to yield an immediate neurological deficit. There is no reported
HoTN that would yield watershed infarcts, and the close proximity to the femur repair would
make diffuse embolic infarcts and PRES less likely.
24. 40F sustained an AComm aneurysm rupture 48h ago. She now has Na 130. Assessment of
which of the following parameters is most appropriate before correction with an infusion of 100
mL/hr of sodium chloride 0.9%?

-Urine Na
-Fractional excretion of uric acid
-Serum Osm
-Volume status
-Urine Osm
Volume status. The initial evaluation of hypoNa is to distinguish true hypoosmolar hypoNa from
translocational hypoNa (mannitol, hyperglycemia, uremia, ethanol) or pseudohyponatremia (eg
hyperproteinemia, dyslipidemia) via SOsm. A normal or elevated SOsm suggests either
translocational hyponatremia or pseudohypoNa. Both CSW and SIADH are d/t abnormal water
or sodium excretion, and therefore do not have inappropriately high urine osmolality. However,
both SIADH and CSW have elevated urine sodium, and therefore, cannot be distinguished based
on this finding alone. SIADH and CSW are differentiated by volume status. SIADH is associated
with a euvolemic or slightly hypervolemic state, whereas CSW results in a hypovolemic state.
Fractional excretion of uric acid (FeUA) may have some clinical utility. Initially, FeUA is
elevated (> 10) in both CSW and SIADH. With normalization of sodium, FeUA corrects in
SIADH, however remains elevated in CSW. Until significant hypovolemia develops, patients
with CSW tend to have higher urine volumes and high 24 hour urinary excretion of sodium.
Alternatively in SIADH, urine volume is normal to low, and therefore sodium excretion is also
normal to low. As distinguishing mild hypovolemia from euvolemia is challenging, this feature
may have greater practical use at the bedside.
24. (Bank #1) 23M involved in MVC exhibits sudden neurological decline from initial GCS 15
to 9. He has a new left hemiparesis with a right dilated pupil. Which of the following is the most
appropriate next step in mgt of this patient?

-Noncontrast head CT
-Administration of mannitol
-Assessment of respiratory status
-Aspirin for evolving stroke
-Craniotomy
Assessment of respiratory status. This patient is developing cerebral herniation syndrome with
likely expanding intracranial mass lesion. Although mannitol, imaging, and likely eventual
surgical intervention are part of the patient’s management, the first intervention involves the
“ABCs” (airway, breathing, and circulation). This includes obtaining a secure airway and
well as hemodynamic stability prior to the next stage of the patient’s treatment.
25. 20M in septic shock has SBP of 80mmHg. Infusion of which of the following is
recommended to increase his BP and improve cardiac output?

-Vasopressin
-Epinephrine
-Amiodarone
-Norepinephrine
-Dopamine
Norepinephrine is the 1st line vasopressor in shock and is associated with a lower mortality rate
as well as fewer adverse effects. Dopamine has similar actions but is associated with
significantly more tachydysrhythmias and should be reserved for patients with bradycardia.
Epinephrine and vasopressin are appropriate 2nd line vasopressors and may enable use of lower
doses of norepinephrine while improving hemodynamics. Inotropes may be added in patients
with cardiac dysfunction.
26. 46F is recovering 5d s/p clipping of ruptured ACommA aneurysm that presented with World
Federation of Neurosurgical Societies (WFNS) grade 1 SAH. She is neurologically intact.
Routine management of this patient in the ICU includes which of the following?

-Prophylactic cerebral angioplasty


-Oral nimodipine
-Hemodilution to hematocrit of 33
-Maintaining SBP < 140
-Daily TCDs
Oral nimodipine has been shown in RCT to reduce the risk of delayed ischemic events after
aSAH. It is common practice to keep the SBP < 160 mmHg prior to securing a ruptured
aneurysm. Once the aneurysm is secured, permissive HTN rather than anti-HTN therapy is
common practice. Patients without vasospasm should be kept euvolemic. Prophylactic
angioplasty is not recommended for patients with aSAH, regardless of the grade.
27. Which of the following is the most likely explanation for demyelination in multiple
sclerosis?

-Developed of gray matter plaques


-Increased vitamin D
-Hyperactivity of the innate immune system
-Increased sun exposure
-T cell sensitization to a component of myelin
T cell sensitization to a component of the myelin is thought to trigger demyelination.
Prominent components include myelin oligodendrocyte glycoprotein (MOG) and myelin basic
protein (MBP). T cells have been demonstrated to initiate the MS plaques. MS plaques occur in
the white matter. While both B and T cells are involved in the pathophysiology of MS, the innate
immune system is not suspect to play a prominent role in the occurrence of demyelination.
Vitamin D deficiency and decreased sun exposure are thought to contribute to MS and may help
to explain why MS is more prominent in populations living at higher altitutes.
28. Multiple endocrine neoplasia (type I) syndrome is characterized by tumors of the pituitary
gland and the

-Pancreas
-Kidneys
-Spinal column
-Brainstem
-Heart
Pancreas. MEN type I is a hereditary condition that leads to tumors of endocrine glands. The
most common tumors are of the pituitary gland, pancreas, and parathyroid. The other lesions are
not typically seen in this condition.

29. If a patient on dabigatran presents with a post-traumatic aSDH requiring surgery, which of
the following drugs can be used to reverse the anticoagulation of dabigatran?

-Idarucizumab
-PCC
-Vit K
-TXA
-FFP
Idarucizumab. Many agents have been administered for the reversal of dabigatran, but the only
effective medication is idarucizumab. This is a monoclonal Ab fragment specifically designed to
reverse the anticoagulation effects. The other medications may have some effect but are not the
appropriate therapy in an operative intracranial mass lesion.
30. A patient who has severe asthma requires administration of an anti-HTN drug. Which of the
following drug classes is contraindicated in this patient?

-Non-selective beta-blockers
-Calcium channel blockers
-Angiotensin receptor blockers (ARBs)
-ACE inhibitors
-Diuretics
Non-selective beta-blockers should not be prescribed for the mgt of comorbidities in patients
with asthma while cardio-selective B-blockers, preferably in low doses, may be used when
strongly indicated and other therapeutic options are not available. There are no contraindications
to use of diuretics, ACE inhibitors, or ARBs in patients with asthma.
31. In which of the following situations are corticosteroids indicated for the treatment of sepsis?

-SBP < 100 refractory to IV fluids and vasopressor administration


-SBP > 100 on multiple abx
-SBP > 100 with IV fluids
- SBP < 100 with known bacterial infxn
- SBP > 100 requiring low dose vasopressors
SBP < 100 refractory to IV fluids and vasopressor administration. Corticosteroid use in septic
shock is indicated as adjunctive therapy for hemodynamic support for patients needing escalating
doses of vasopressors after optimal fluid resuscitation. However, it is not a 1st line treatment of
HoTN in septic shock. In adults with septic shock treated with low dose corticosteroids, short-
and longer-term mortality are unaffected, adverse events increase, but duration of shock,
mechanical ventilation and ICU stay are reduced. The etiology of the septic shock and
concurrent use of abx are variables that have no relevance to the concurrent use of
corticosteroids.
32. Which of the following is most likely to be decreased in a patient who has pulmonary
edema?

-PaO2
-Central venous pressure (CVP)
-Vascular permeability
-Pulmonary capillary wedge pressure
-Hydrostatic pressure
PaO2.
Discussion:
Normal pulmonary physiology favors a small net influx of fluid from the alveolar capillaries into
the lung interstitial space. The amount of fluid leaking from capillaries is dependent on the
balance b/w hydrostatic and osmotic pressure. Pulmonary edema can occur d/t a cardiogenic
etiology characterized by an increased hydrostatic pressure or noncardiogenic etiologies
characterized by increased vascular permeability. In cardiogenic pulmonary edema, pulmonary
capillary wedge pressure (PCWP), which is an estimate of left atrial filling pressure is increased.
Cardiogenic pulmonary edema is also characterized by increased central venous pressure (CVP).
References:
1. N Engl J Med. 2005 Dec 29;353(26):2788-96. 2. Crit Care Clin. 2015 Oct;31(4):803-21

33. 30M with hx of alcohol abuse is admitted to the neurointensive care unit with AMS
following mild head trauma. Serum sodium level is 109 mEq/L; 3% sodium chloride is initiated.
Over the next 36h, Na level normalizes, but mental status deteriorates, and the patient becomes
dysarthric and quadriparetic. This patient's deterioration is most likely due to which of the
following?

-Alcohol withdrawal
-Osmotic demyelination
-Arterial vasospasm
-Thiamine deficiency
-Epileptic seizure
Osmotic demyelination. The patient is experiencing neurologic deterioration d/t rapid over-
correction of acute symptomatic hypoNa, leading to osmotic demyelination syndrome
characterized by delayed neurologic deterioration after 2-6 days of sodium correction. Symptoms
consist of dysarthria, dysphagia, paraparesis/quadriparesis, seizures, lethargy, coma. In this
scenario, sodium correction must be promptly, but cautiously, corrected up to 4-6 mEq/L
(maximum 8mEq/L) in a 24 hour period. Arterial spasm and epileptic seizures would not be
expected in mild TBI, given the context of sodium correction. Alcohol withdrawal would be
associated with tremors, diaphoresis and hemodynamic instability. Thiamine deficiency is
possible but does not follow the temporal course of the over-correction of hyponatremia and
associated neurologic deterioration.
33. (Bank #1) The American Heart and American Stroke Associations recommend that
intravenous tissue plasminogen activator (tPA) be administered no later than how long after the
onset of acute ischemic stroke?

-4.5h
-24h
-12h
-2h
-6h
4.5h. Administration of IV tPA is a mainstay in the management and treatment of patients with
acute ischemic stroke. Initial studies demonstrated benefit for intravenous tPA given 0-3 hours
after stroke onset. A later publication demonstrated utility of tPA up to 4.5 hours after stroke
onset. Numerous mechanical thrombectomy trials for patients with large vessel occlusion acute
ischemic strokes have demonstrated efficacy in both "early" and "late" windows, expanding the
indication for this intervention to 24 hours. However, this question pertained specifically to
recommendations regarding intravenous tPA administration.
34. (Bank #1) Which of the following parameters should be confirmed before a formal apnea test
can be performed?

-PaO2 is normal or elevated


-Absence of deep tendon reflexes
-No uptake on nuclear medicine perfusion test
-Non-responsive EEG
-Patient has no autonomic responses (flushing, sweating)
PaO2 is normal or elevated. The apnea test is generally the last test performed to determine brain
death. After confirming the patient is in an irreversible coma and that there are no brainstem
reflexes, the patient is pre-oxygenated, taken off the ventilator, and observed for evidence of
spontaneous breathing. If the patient remains apneic when the PaCO2 reaches ≥ 60 mm Hg (or
20 mm Hg over baseline) the patient is declared brain dead. Prior to initiating the test it must be
confirmed that the PaO2 is normal and is typical to preoxygenate to a PaO2 > 200. After taking
the patient off the ventilator, oxygen is continuously supplied through a catheter threaded down
the ET tube to ensure the patient is not hypoxic during the test. Ancillary tests such as a nuclear
medicine perfusion test or EEG may be performed when apnea testing is indeterminate but are
not required for the determination of brain death or as a qualification for the apnea test.
Autonomic functions and DTRs are not considered brainstem mediated responses and the
presence of these responses does not exclude the diagnosis of brain death.

35. 57F with grade III SAH becomes progressively obtunded 3d s/p clipping of ACommA
aneurysm. Laboratory studies show a serum Na level of 119 mEq/L, a serum osmolarity of 260
mOsm/L, and a urinary Na level of 40mEq/L. A post-operative CT scan shows no abnormalities.
The most appropriate treatment is IV administration of which of the following agents?

-DDAVP
-23% NS
-2% NS
-IV steroids
-Dextrose 5% water
2% NS. This patient is experiencing neurologic deterioration d/t acute symptomatic hypoNa. In
this scenario, Na correction must be promptly, but cautiously, corrected within a few hours
targeting an increase of 4-6 mEq/L (maximum 8 mEq/L ). In acute symptomatic hypoNa, a
correction of 4-6 mEq/L (up to 8 mEq/L) is safe and well tolerated and can be achieved within a
few hours, with close monitoring and maintenance of levels up to 24h. A more rapid correction
can lead to osmotic demyelination syndrome with neurologic sequelae thereof. A slightly
hypertonic saline solution like 2% normal saline can be administered, since iso-osmotic solutions
can worsen hypoNa by promoting more water retention compared to sodium, potentiating
SIADH. A hypotonic solution like Dextrose 5% can worsen hyponatremia. DDAVP is useful in
diabetes insipidus and thus not in this scenario. A significantly hyperosmolar solution such as
23% normal saline will over-correct the sodium rapidly, which can be dangerous.
36. Which of the following is most likely to result from a jugular venous oxygen saturation of
less than 50%?

-Global cerebral hypoxia


-Barbiturate coma
-Hyperperfusion syndrome
-Posterior fossa ischemia
-ICA-MCA tandem occlusion and stroke
Global cerebral hypoxia.
Discussion:
Jugular venous oxygen saturation is typically measured using a central line catheter inserted
retrograde in the internal jugular vein with the tip in the jugular bulb. Depending on the
equipment used, intermittent or continuous sampling of the oxygen saturation of the blood
exiting the brain via the jugular vein can be measured. Typical measurements range from 50% to
75%. During global cerebral hypoxia, decreased oxygen delivery will cause an increase in
oxygen extraction in the capillary bed and a drop in the jugular venous oxygenation below 50%.
While an ICA-MCA occlusion will certainly cause ischemia, complete absence of flow may not
cause a measurable increase in oxygen extraction as there can be no increase in oxygen
extraction if flow drops to zero. Posterior fossa ischemia will often not result in a change jugular
venous oxygen saturation for two reasons. First, the catheter is usually placed on the dominant
side which is the side typically draining the supratentorial space. Second, the posterior
circulation only accounts for 15-20% of total brain metabolism so its contribution to overall
oxygen extraction is fairly small. Cerebral hyper perfusion syndrome and barbiturate coma
would be expected to result in excess oxygen delivery and therefore an increase in jugular
venous oxygen saturation above 75%.
References:
Ullman JS: Cerebral blood flow and metabolism in Intensive Care in Neurosurgery. Andrews
BT, ed. New York, Thieme, pp29-46, 2003.Schell, Randall M., and Daniel J. Cole. 2000.
“Cerebral Monitoring: Jugular Venous Oximetry.” Anesthesia & Analgesia 90 (3): 559.
37. 56M sustains severe TBI in a MVC. His ICP remains increased despite administration of
hyperosmolar agents, CSF drainage, sedation, and paralysis. A pentobarbital coma is induced.
Which of the following is a potential mechanism of neuroprotection from this intervention?

-Increased blood flow


-Activation of the GABA-B receptor
-Reducing metabolic demand
-Seizure prevention
-Decreased CSF production
Reducing metabolic demand. When GABA binds to the GABA-A receptor on a neuron it causes
opening of chloride channel that results in hyperpolarization and reduced synaptic firing.
Barbiturates such as pentobarbital also bind to the GABA-A receptor, but at a different site than
the GABA molecule itself. The effect of the barbiturate is to keep the chloride channel open
longer, potentiating the inhibitory effect of the GABA molecule, a process referred to as
allosteric modulation. In high doses, barbiturates result in a reduction in synaptic firing. As
synaptic firing accounts for 50% of brain metabolism, barbiturates can significantly reduce
metabolic demand. While barbiturates are used for seizure prophylaxis and treatment, seizure
prevention is not a mechanism of neuroprotection. Barbiturates do not affect CSF production.
Barbiturates do not bind to the GABA-B receptor. One criticism regarding the use of barbiturates
for neuroprotection is their tendency to cause hypotension and reduced cerebral perfusion.
38. Which of the following complications has been linked to aggressive use of pressors and IVF
to maintain elevated CBF in patients with severe TBI?

-Worsening neurological outcome


-ARDS
-Refractory elevated ICP
-AKI
-Progression of intracranial hematoma
ARDS. Robertson et al reported a clinical trial comparing intracranial pressure (ICP) directed
therapy with cerebral perfusion pressure (CPP) directed therapy and found that artificially
elevating the CPP > 70mmHg led to a five-fold increase in the incidence of acute respiratory
distress syndrome. The other variables, although theoretically possible, were not shown to be
true in this trial.
39. A previously healthy 26M is involved in a MVC, resulting in severe head trauma that is c/b
an anoxic brain injury during extrication from the vehicle. It is determined that there is no
medical or surgical intervention that can help this patient. The patient's family would like to
move forward with organ donation. Which of the following would prevent organ donation at this
time?

-Unable to confirm organ donor status on driver's license


-Sepsis
-Patient is not brain dead
-Professional tattoos
-Alcoholism
Sepsis. There are surprisingly few absolute contra-indications to organ donation, but sepsis is
one of them. Alcoholism and (non-prison) tattoos do not typically exclude organ donation.
Families can choose organ donation for family members who do not have their donor status
explicitly stated on their license or proxy documents. Donation after cardiac death (DCD) allows
for donation in patients who are not brain dead.
40. Which of the following age groups is at greatest risk for TBI?

- >=65
- 55-64
- 25-34
- 20-24
- 45-54
>=65.
Discussion:
Children aged 0 to 4 years, older adolescents aged 15 to 19 years, and adults aged 65 years and
older are most likely to sustain a traumatic brain injury.
References:
1. U.S. Centers for Disease Control. Incidence Rates of Hospitalization Related to Traumatic
Brain Injury --- 12 States, 2002. 2. https://www.aans.org/Patients/Neurosurgical-Conditions-and-
Treatments/Traumatic-Brain-Injury
41. SIADH secretion is supported by the findings of decreased serum Na level accompanied by
which of the following?

- High serum Osm


- Low urine Osm
- Low serum Osm
- High serum Na
- High TSH
Low serum Osm. SIADH is characterized by lab values consistent with low TSH levels, high
urine osmolality and low serum osmolality due to water retention. High Urine Na would be
expected in cerebral salt wasting. SIADH is associated with low serum Na.
42. In the management of severe traumatic brain injury, when is hyperventilation therapy
indicated?

-When brain tissue oxygen levels are low


-Never
-In conjunction with barbiturate coma
-Only as a temporizing measure
-Within the first 24h
Only as a temporizing measure. There is no level I evidence regarding the use of
hyperventilation. According to the Guidelines for the Management of Severe Traumatic Brain
Injury, Fourth Edition, hyperventilation should only be used as a temporizing measure such as en
route to operating room or while awaiting other interventions. It should be avoided during the
first 24hrs after injury as this can lead to further vasoconstriction and decrease of cerebral
blood flow.
43. Which of the following best characterizes Cheyne-Stokes respirations?

-Sustained hyperventilation
-Irregular, gasping respiration
-Apnea
-Periodic breathing with phases of hyperpnea alternating with apnea
-Respiratory pause at full inspiration
Periodic breathing with phases of hyperpnea alternating with apnea. Cheyne-Stokes respiration is
characterized by phases of hyperpnea alternating with apnea. Respiratory depth during the
hyperpneic phase increases from breath to breath in a crescendo until a peak is reached followed
by a decrescendo. A period of apnea follows which usually lasts 10-20 seconds. Cheyne-Stokes
respiration is seen in metabolic encephalopathies and with lesions that impair forebrain and
diencephalon function. Apnea occurs when lesions affect the ventral respiratory group in the
ventrolateral medulla bilaterally. Irregular gasping breathing characterizes cluster or ataxic
breathing, which is seen with lesions of the pontomedullary junction. A respiratory pause at full
inspiration characterizes apneusis, which is seen with bilateral pontine lesions. Sustained
hyperventilation is seen with metabolic encephalopathies and rarely in cases of high brainstem
tumors.
44. ECG changes observed in patients with severe hyperK include which of the following?

-ST-segment depression & increased PR interval


-Shortened QT interval & widened QRS complex
-None of the above
-All of the above
-Peaked T-waves
All of the above. EKG changes secondary to hyperkalemia include peaked T waves, shortened
QT interval, and ST-segment depression. These changes are followed by bundle-branch blocks
causing a widening of the QRS complex, increases in the PR interval and decreased amplitude of
the P wave.
45. 70F with hx of poorly controlled HTN and T2DM p/w 2-cm L thalamic hemorrhage. She has
right hemiparesis but is currently awake and following commands on the L side. Blood pressure
on presentation is 200/100 mmHg. Immediate management of this patient should include which
of the following?

-Administration of steroids
-Blood pressure reduction
-Emergent intubation
-MRI brain
-EEG
Blood pressure reduction. Patients with acute ICH need close medical attention in the initial 24-
48 hours involving airway, breathing, circulatory support, after which BP control and reversal of
coagulopathy is essential. The role of other diagnostic modalities in the acute settings is then
considered based on clinical history. This patient is awake and alert, not requiring emergent
intubation. There is no role for steroids as a therapeutic modality in spontaneous ICH.
46. Postoperative hypocalcemia is most likely to have which of the following cardiovascular
effects?

-Torsades de pointed (polymorphic ventricular tachycardia)


-Heart failure
-Prolonged QT interval on EKG
-Acute cardiac ischemia
-HoTN
Prolonged QT interval on EKG. HoTN may complicate acute hypocalcemia, particularly when
rapidly induced by transfusion of citrated blood or with use of low calcium dialysate in the
patients undergoing renal replacement therapy. Heart failure has been reported in severe cases
but is not the most common occurrence. Hypocalcemia characteristically causes prolongation of
the QT interval on EKG. Hypocalcemia prolongs phase 2 of the action potential with the impact
modulated by the rate of change of serum calcium concentration and function of the myocyte
calcium channels. Torsades de pointes can be triggered by hypocalcemia but is much less
common than with hypokalemia or hypomagnesemia.
46. 20M exhibits a reduction in his ICP when his arterial BP rises. This is an example of which
of the following physiologic phenomena?

-Hypertensive crisis
-Intact cerebral autoregulation
-Defective cerebral autoregulation
-Cushing's response
-Shock
Intact cerebral autoregulation.
Discussion:
Cerebral autoregulation mediates changes to cerebral blood volume and, in turn, changes in ICP.
If autoregulation is intact, decreases in CPP result in vasodilation. This causes increased CBV
and increased ICP. Conversely, increases in CPP result in vasoconstriction at the level of the
largest arterioles which causes decreased CBV. This leads to a reduction in ICP. Neither
Cushing's response, hypertensive crisis or shock are appropriate answers for this phenomenon.
References:
Rangel-Castilla L, Gasco J, Nauta HJ, et al. Cerebral pressure autoregulation in traumatic brain
injury. Neurosurg Focus. 2008 Oct;25(4):E7.Ter Minassian A, Dubé L, Guilleux AM, et al.
Changes in intracranial pressure and cerebral autoregulation in patients with severe traumatic
brain injury. Crit Care Med. 2002 Jul;30(7):1616-22
47. 42M w/ a head injury has clear nasal drainage. Which of the following is the most useful
method to determine the nature of this fluid?

-CT of the brain and sinuses


-Observation over time
-Send fluid for beta-2 transferrin
-Lumbar puncture
-Endoscopic evaluation
Send fluid for beta-2 transferrin. The patient's condition is c/f CSF leakage. The best method to
differentiate CSF from normal discharge is beta-2 transferrin, which would be seen in CSF and
not typical nasal fluid. Endoscopic evaluation, imaging, and observation would be suggestive but
not diagnostic. Lumbar puncture would be useful to r/o meningitis.
49. 48h s/p clipping of a HH3 ruptured aneurysm with intraoperative lumbar spinal drainage and
lamina terminalis fenestration, a 56F becomes somnolent with pinpoint pupils and flexure
posturing. Immediately after the procedure, she was oriented to person and place. CT scan shows
small ventricles and crowding of the brain stem. Which of the following is the most appropriate
initial step in management?

-Position patient in Trendelenburg position


-Insert a right frontal ventricular drain
-Give mannitol 1g/kg IV over 30 minutes
-Return to OR for decompressive craniotomy
-Initiate HHH therapy
Position patient in Trendelenburg position. This pt is suffereing from central herniation
syndrome d/t over-drainage of spinal fluid from the lumbar drain. The first step is to put the
patient in reverse Trendelenburg position. A ventricular drain in this setting is used to treat non-
communicating HCP which is unlikely considering that the ventricles are small and the lamina
terminalis has been fenestrated. HHH therapy is used for the mgt of vasospasm. Brain swelling
requiring mannitol or return to OR is in the differential but the patient is a bit early for severe
vasospasm and a bit late for post-operative edema.
50. 23F is admitted to the ICU s/p unrestrained high-speed MVC. Pulse is 44/min and regular,
respirations are 16/min, BP is 68/40 mmHg, and central venous pressure is 2 mmHg. Fluid
resuscitation is initiated, but the patient remains hypotensive and bradycardic. Which of the
following is the most likely cause of shock and the most appropriate treatment?

-Cardiogenic Shock- sympathomimetic vasopressors + fluid replacement


-Obstructive Shock- immediate causal treatment
-None of the above
-Distributive Shock- sympathomimetic vasopressors + fluid replacement
-Hypovolemic Shock- fluid resuscitation (balance crystalloids)
Distributive Shock - sympathomimetic vasopressors + fluid replacement

Discussion:
This patient most likely is in a state of neurogenic shock, characterized by bradycardia,
hypotension, and low central venous pressures. Neurogenic shock is a state of imbalance
between sympathetic and parasympathetic regulation of cardiac action and vascular smooth
muscle. The dominant signs are profound vasodilation with relative hypovolemia while blood
volume remains unchanged, at least initially. Neurogenic shock is classified as a type of
distributive shock. The primary treatment of neurogenic/distributive shock is fluid resuscitation
(typically balanced crystalloids) and administration of sympathomimetic (norepinephrine,
epinephrine) vasoactive medications.Hypovolemic shock is a condition of inadequate organ
perfusion caused by loss of intravascular volume, usually acute. Early hypovolemic shock is
typically characterized by tachycardia and hypotension and low central venous
pressures.Cardiogenic shock is primarily a disorder of cardiac function in the form of a critical
reduction of the heart's pumping capacity, caused by systolic or diastolic dysfunction leading to a
reduced ejection fraction or impaired ventricular filling. Patients with cardiogenic shock may be
either tachycardic or bradycardic, however, central venous pressures are usually high.Obstructive
shock is a condition caused by the obstruction of the great vessels or the heart itself. Although
the symptoms resemble those of cardiogenic shock, obstructive shock needs to be clearly
distinguished from the latter because it is treated quite differently. The treatment of obstructive
shock is causal - thrombolysis of pulmonary embolism, tension pneumothorax or cardiac
tamponade by thoracic/pericardial drainage. Central venous pressures would be expected to be
high in ob
51. The randomized controlled Decompressive Craniectomy in Patients with Severe TBI
(DECRA) trial concluded that patients who received a craniectomy for severe TBI injury
experienced which of the following?

-Fewer complications
-Higher mortality
-Decreased ICP
-Better functional outcomes
-No benefit
Decreased ICP. The DECRA trial randomized patients to continued medical mgt (pentobarbital
coma) or surgical decompression of the patient was refractory to most ICP measures. The study
found that patients who had surgical decompression had fewer episodes of elevated ICP and
fewer days in the ICU. They had similar mortality to the medical group but unfavorable
outcomes. Criticisms of this trial including failure of randomization as well as too early to
decompression in the surgical group.
53. (Bank #1) 25M injured in MVC is admitted to the ICU with a GCS of 6 and CT scan
demonstrating a 2cm L temporal contusion and scattered tSAH. Medical evidence-based clinical
data support a 1-week course of ppx with a drug from which of the following classes?

-Anticonvulsants
-Barbiturates
-Antibiotics
-Steroids
-Hyperosmolars
Anticonvulsants. Post-traumatic sz (PTS) occur either early (within 7d of injury) or late (after
7d). Clinical PTS occur in up to 12% of patients after severe TBI with electrographic sx
occurring in up to 20-25% of cases. Anticonvulsant therapy is recommended to decrease the
incidence of early PTS but not late PTS. Results from the CRASH trial provided Level I
evidence that the use of steroids after TBI were associated with increased mortality. Therefore,
the use of steroids after TBI is currently not recommended for improving outcomes or reducing
ICP. Current guidelines do not recommend prophylactic use of antibiotics after TBI. While a
study of 100 critically ill patients (86%) with TBI showed a decreased risk of pneumonia with
cefuroxime treatment after intubation, no mortality benefit was seen and use of prophylactic
antibiotics may contribute to development of resistant organisms (Am J Respir Crit Care Med.
May 1997;155(5):1729-1734. PMID: 9154884). Prophylactic use of barbiturates after TBI is not
recommended, although barbiturates may be used to control refractory increases in ICP.
Hyperosmolar therapy is a key medical therapy for treating increased intracranial pressure,
however there is no current indication for prophylactic use of hyperosmolar therapy.
54. 35M comes to ED because of 2d hx of severe H/A. The CT scan shown is obtained. The
patient is awake, alert, and conversing, but when the patient is returned from the CT scan, he is
acutely unresponsive hypertensive, and has bradycardia. Which of the following is the most
appropriate next step in mgt?

-Have the patient intubated


-Administer atropine
-Place an EVD
-Repeat head CT
-Start IV antihypertensive
Have the patient intubated. The patient has likely suffered rupture (in this case likely rerupture)
of a cerebral aneurysm. He is experiencing Cushing's triad (unresponsive, hypertensive
bradycardic) due to elevated intracranial pressure. The patient is unresponsive and should be
intubated. This will also allow for mild hyperventilation to help reduce ICP. The next steps
include CT scan and placement of an EVD, and intravenous antihypertensives if blood pressure
remains above 160. As the bradycardia is likely caused by the severe hypertension and the
patient is not hypotensive, atropine would not typically be administered.
55. The Transfusion Requirments in Critical Care (TRICC) trial demonstrated that liberal
transfusion (Hgb < 10) vs restrictive transfusion (Hgb 7) protocols are associated with which of
the following outcomes in critically ill patients?

-Higher rates of multi-organ failure in the restrictive-strategy transfusion group


-Higher rates of multi-organ failure in the liberal-strategy transfusion group
-The mortality rates during hospitalization were higher in the liberal transfusion group
-Cardiac events (pulmonary edema, myocardial infarction) were more frequent in the restrictive-
strategy transfusion group in patients in the ICU
-The rate of death within 30 days of admission were similar between the liberal and restrictive-
strategy transfusion groups
The rate of death within 30 days of admission were similar between the liberal and restrictive-
strategy transfusion groups.

Discussion:
The primary outcome — the rate of death from all causes in the 30 days after admission to the
intensive care unit — was 18.7 percent in the restrictive-strategy group and 23.3 percent in the
liberal-strategy group (95 percent confidence interval for the difference between the groups, -
0.84 percent to 10.2 percent; P=0.11) The mortality rates during hospitalization were lower in the
restrictive-strategy group (22.2 percent vs. 28.1 percent, P=0.05). Other mortality rates including
the mortality rate during the entire stay in the intensive care unit (13.9 percent vs. 16.2 percent,
P=0.29) and the 60-day mortality rate (22.7 percent vs. 26.5 percent, P=0.23) were also lower in
the restrictive-strategy group but not significantly so.The number of patients with multiorgan
failure (more than three organs), which was analyzed as a dichotomous variable (present or
absent) for each of seven organ systems,18 was not significantly different between the
restrictive-strategy and liberal-strategy groups (5.3 percent vs. 4.3 percent, P=0.36).Cardiac
events, primarily pulmonary edema and myocardial infarction, were more frequent in the liberal-
strategy group than in the restrictive-strategy group during the stay in the intensive care unit
(P<0.01).The mortality rates during hospitalization were lower in the restrictive-strategy group
(22.2 percent vs. 28.1 percent, P=0.05).

References:
Hébert PC, Wells G, Blajchman MA, Marshall J, Martin C, Pagliarello G, Tweeddale M,
Schweitzer I, Yetisir E. A multicenter, randomized, controlled clinical trial of transfusion
requirements in critical care. Transfusion Requirements in Critical Care Investigators, Canadian
Critical Care Trials Group. N Engl J
56. 60M is admitted to the ICU after sustaining an inoperative closed head injury in a MVC. Lab
studies show an increased serum creatinine level, muddy brown casts in urine sediment, and an
increased fractional excretion of sodium. Which of the following is the most likely diagnosis?

-Dehydration
-Acute tubular necrosis (ATN)
-Acute bladder obstruction
-Kidney laceration
-Cerebral salt wasting
ATN. Muddy brown epithelial casts in the urine are pathognomonic for acute tubular necrosis
(ATN). The casts form when either ischemia or toxins cause damage to the epithelial cells of the
renal tubules causing them to slough off and form casts in the renal tubule. The casts are then
eventually excreted into the urine. ATN can be caused by ischemia (usually due to shock) or
toxins.The fractional excretion of sodium is the percentage of sodium that is excreted into the
urine during renal filtration. Intrinsic kidney failure such as ATN will result in an increased
FENa as sodium recovery in the renal tubule is impaired. Extrinsic causes of renal failure such as
dehydration or acute bladder obstruction will typically have a low FENa, at least in the acute
phase. A kidney laceration would cause blood in the urine. Cerebral salt wasting can be caused
by a closed head injury and may be associated with an elevated FENa. The cause of CSW is not
well understood but has been attributed to perturbations in the sympathetic nervous system or the
release of BNP. It is not, however, a kidney injury in itself and is not associated with casts in the
urine unless there is secondary injury to the kidney due to severe hypovolemia.

57. A decrease in which of the following explains the mechanism by which hyperventilation
lowers ICP?

-Oxygen saturation
-PaCO2
-Mean arterial pressure
-PEEP
-Arteriolar smooth muscle tone
Hyperventilation causes decreased PaCO2 which incites constriction of arteriolar smooth
muscles (increased tone). This results in decreased cerebral blood flow and decreased cerebral
blood volume which leads to a decrement in ICP.
58. Which of the following treatments is most likely to decrease the incidence of chronic
subdural hematoma recurrence?

-Induced hyponatremia
-Irrigation of the subdural space
-Hyperoxygen therapy
-Placement of subdural drain
-Bedrest with head of bed flat
Placement of subdural drain. In a randomized trial by Sartarius et al (2009), the placement of a
SDD at the time of surgery decreased the incidence of symptomatic recurrence. All of the other
methods have been utilized to try to decrease the incidence of recurrence after surgery. None has
been shown to be effective in rigorous study. Newer techniques such as MMA embolization are
currently being studied.
57. (Bank #1) Which of the following treatments is most appropriate for a patient with a
hemolytic transfusion reaction?

-Start diuretics
-Emergent plasmapheresis
-Administration of Ringer's lactate
-Emergent steroid administration
-Clinical assessment and hemodynamic support
Clinical assessment and hemodynamic support. Pts with transfusion reactions promptly need
medical attn, which initially involve assessment of ABCs. First the txfn should be stopped while
preserving venous access. Pts may respond to oxygen administered by nasal catheter or mask,
but they may need to be intubated for mechanical ventilation. Without signs of volume overload
or cardiogenic pulmonary edema, diuretics and plasmapheresis are not indicated. No evidence
exists that corticosteroids or antihistamines are beneficial. Treat complications with specific
supportive measures.
57. According to the results of a prospective, randomized, controlled trials, which of the
following best describes the effect of therapeutic hypothermia in pts with a severe TBI?

-Attenuation of the release of excitatory neurotransmitters following severe TBI


-Insufficient evidence for its utilization in the mgt of pts with severe TBI
-Decreased incidence of myocardial ischemia
-Improved GOS-E at 30 days
-Decreased incidence of venous thromboembolic complications
Despite a large # of studies, there remains no high-quality evidence that hypothermia is
beneficial in the treatment of patients with severe TBI. While hypothermia may be associated
with a decreased release of excitatory neurotransmitters (which, along with reducing cerebral
metabolic rate, may explain its MOA) there is insufficient evidence to show that therapeutic
hypothermia improves mortality or long-term functional outcomes. The main risks associated
with therapeutic hypothermia include increased risk of infectious complications, coagulation
abnormalities, MI, and A-Fib.
58. 49M is brought to ED s/p MVC. Physical exam demonstrates no verbal response, eye
opening only to painful stimuli, and localizing in response to noxious stimuli. Which of the
following is the most likely Glasgow Coma Scale score in this patient?

-7
-10
-6
-8
-11
The Glasgow Coma Scale (GCS) score is made up of three components (Eye, Verbal, and
Motor) for a total score of 3 to 15. This particular patient has a score of E2 (eyes open to pain),
V1 (no verbal response), and M5 (localization) for a total score of 8.
58. Which of the following is the initial fluid of choice for resuscitation of patients in septic
shock?

-LR
-Concentrated 25% albumin
-Pentastarch
-Hydroxyethyl starch
-Hypertonic saline
The ideal fluid of choice in the resuscitation of patients in septic shock is a isotonic solution,
based on several clinical trials. There is increasing evidence that normal saline is associated with
increased mortality and kidney injury; balanced isotonic crystalloids may be a safer alternative.
Hyperosmolar solutions have been associated with worse outcomes. Administration of Albumin
5% can be considered, although controversial, with lesser clinical evidence. There is no evidence
for the efficacy for concentrated Albumin 25%. Hydroxyethyl starches appear to increase
mortality and kidney injury in the critically ill and are no longer indicated in these patients.

59. HIGH YIELD. 63M sustains a hemorrhagic p-fossa stroke and is taken to the OR for
emergency surgical evacuation. the patient is placed in the lateral position for the procedure.
Shortly after elevating the bone flap, the end-tidal CO2 and O2 saturation decrease, leading to a
decrease in the patient's blood pressure. Which of the following is the most appropriate course of
action?

-Immediately close the wound and prepare for reintubation


-Resect cerebellar hemisphere to relieve pressure
-Immediate place an EVD at Frazier's point
-Flood the field with saline
-Increase PEEP
Flood the field with saline. A drop in ET-CO2 and O2 saturation during posterior fossa
surgery likely represents a venous air embolus due to air entering non-compressed veins
such as the dural sinuses and bony venous lakes during the craniotomy. The first step for the
surgeon is to flood the field with saline and/or cover the field with saline soaked sponges to help
seal the veins off from the air. Posterior fossa swelling requiring tissue resection or a ventricular
drain would not likely cause significant changes in the oxygenation status of a mechanically
intubated patient. Increasing PEEP might be considered initially but is not likely to be effective
as the air embolus affects perfusion not ventilation. Disconnection of the ventilator or
dislodgment of the ET tube might result in a drop in ET-CO2 and O2 sat, but would not likely
cause hypotension, at least initially.
60. Which of the following agents is most likely to partially reverse the effects of enoxaparin?

-Tranexamic acid
-Protamine
-Platelets
-FFP
-4-factor PCC
Protamine. Heparinoids and their low molecular weight derivatives affect the clotting cascade by
directly and indirectly affecting thrombin and factor X. Enoxaparin represents a commonly used
low molecular weight heparin agent which is partially reversible. Protamine allows for reversal
of 60-80% of the effects of enoxaparin.In order to reverse enoxaparin, it is important to note
when the last dose of the medication was administered. If given within the prior 8 hours, 1mg of
protamine must be administered per 1mg of enoxaparin over 10 minutes, with a maximum dose
of 50mg of protamine. If the enoxaparin was given >8 hours prior, one can give ½ of the
calculated protamine dose.
61. (Bank #4) Which of the following pairs are the key factors that determine the total energy
expenditure of spontaneous ventilation?

-Ventilation and perfusion


-Flow and PEEP
-Pulmonary compliance and resistance
-Alveolar surface area and diffusion
-Alveolar-arterial gradient
Pulmonary compliance and resistance.
Energy expenditure during breathing is directly related to work of breathing (WOB). Work is the
product of pressure and volume. In respiratory physiology, WOB is typically associated with
inspiraotry effort, as long as expiration remains a passive process. WOB can be determined by
calculating the area under the curve of a pressure-volume plot. In this case, pressure is the sum of
the transpulmonary pressure gradient and the chest wall pressure gradient. Alveolar surface area
and diffusion are critical for gas exchange but not directly relevant to the work of breathing. The
work to move air into and out of the lungs plus the work to expand the chest wall is the total
work of breathing. Pulmonar compliance is directly related to WOB as it is the willingness of the
lungs to distend described as the change in volume divided by change in pressure. Airway
pressure during inflation is influenced by volume, thoracic compliance, and resistance to flow.
Pressure gradients allow for flow to be generated, but this is not directly related to calculating
WOB. Positive end expiratory pressure (PEEP) can be intrinsic or extrinsic. PEEP is an
expiratory measurement and does not directly affect WOB. Ventilation and perfusion describe
blood flow and airflow to the lungs but are not directly involved in WOB. The A-a graident
measures the differencxe between oxygen concentration in the alveoli and arterial system. It is
not directly related to WOB but is important in determining the diferential diagnosis for
hypoxemia.
References:
1. Magalhães, P., Padilha, G., Moraes, L. et al. Effects of pressure-support ventilation with
different levels of positive end-expiratory in a mild model of acute respiratory distress syndrome.
ICMx 3 (Suppl 1), A573 (2015). https://doi.org/10.1186/2197-425X-3-S1-A573
2. Grinnan, D.C., Truwit, J.D. Clinical review: Respiratory mechanics in spontaneous and
assisted ventilation. Crit Care 9, 472 (2005). https://doi.org/10.1186/cc3516
62. (Bank #2) Which of the following is the earliest sign of malignant hyperthermia?

-Rising end-tidal CO2


-Hyperthermia
-Metabolic acidosis
-Muscle rigidity
-Hyperkalemia
Rising end-tidal CO2.
Discussion:
Malignant hyperthermia (MH) is an autosomal dominate disorder associated with the ryodine
receptor gene that can result in severe, potentially life-threatening complications with certain
anesthetics. In individuals with a predisposition for MH, exposure to halogenated anesthetics (i.e.
sevoflurane, halothane, enflurane, etc.) or succinylcholine (depolarizing paralytic) results in an
uncontrolled release of calcium from skeletal muscle resulting in sustained muscle contraction.
This results in a depletion of ATP and subsequent increase in O2 consumption and CO2
production. The earliest sign of MH is a rapid, unexplained rise in end-tidal CO2 associated with
persistent tachycardia. Additional symptoms of MH include tachypnea, hypotension, rigidity,
and fever. Laboratory evaluation may reveal anion gap metabolic acidosis, hyperkalemia, and
hypercarbia.
If left untreated, MH can result in cardiac dysrhythmias and cardiac arrest. The treatment for
suspected MH is removal of the offending anesthetic agent, hyperventilation with 100% O2, and
administration of dantrolene. Dantrolene is a muscle relaxant that acts by inhibiting the release of
calcium from the sarcoplasmic reticulum of skeletal muscle.
References:
Watt S, McAllister RK. Malignant Hyperthermia. [Updated 2021 Jul 25]. In: StatPearls
[Internet]. Treasure Island (FL): StatPearls Publishing; 2021 Jan-. Available from:
https://www.ncbi.nlm.nih.gov/books/NBK430828/
Adnet PJ, Gronert GA. Malignant hyperthermia: advances in diagnosis and management. Current
Opinion, Anesthesiology. 1999; 12:353-358https://pubmed.ncbi.nlm.nih.gov/17013338/

63. (Bank #2) A 40-year-old woman is being treated with pressure-support mechanical
ventilation following a severe subarachnoid hemorrhage. Close monitoring of which of the
following is most likely to assure adequate minute ventilation?

-Inspiratory pressure and positive end-expiratory pressure


-Respiratory rate and positive end-expiratory pressure
-Tidal Volume
-Tidal volume and respiratory rate
-Inspiratory pressure and respiratory rate
Inspiratory pressure and respiratory rate

The minute ventilation is the amount of air a person breaths in a minute. The minute ventilation
is calculated by the multiplication of the tidal volume and the respiratory rate. On pressure-
support mode of ventilation, minute ventilation is adjusted by changes in inspiratory pressure and
based on the respiratory rate that the patient is triggering. The tidal volume is not manually
adjustable on pressure-controlled modes of ventilation. A person requires a minimum of 6 to 8
litres of minute volume for the proper oxygenation of the tissues and the removal of carbon
dioxide from the lungs. The minute volume increases at times of stress and exercise. This
increase compensates for the increase in the demand of oxygen and the increased production of
carbon dioxide, usually by increase in respiratory rate.
64. Which of the following describes the mechanism of action of remifentanil?

-Kappa opioid receptor antagonist


-Mu opioid receptor antagonist
-Mu opioid receptor agonist
-GABA-A receptor agonist
-Kappa opioid receptor agonist
Mu opioid receptor agonist

Remifentanil's is as an agonist of the mu opioid receptor. It is a short acting agent used for
sedation and general anesthesia.
Butorphanol and Nalbuphine are kappa opioid receptor agonists used for analgesia. Naltrexone is
an antagonist for mu opioid receptors but less effective for kappa opioid receptors and used to
reverse effects of opioid medications.
Benzodiazepines are GABA-A agonists.
65. (Bank #2) In which of the following types of edema is the extracellular fluid volume
decreased?

-Ionic (Osmotic) Edema


-Amyloid
-Vasogenic
-Interstitial
-Hydrostatic
Ionic (Osmotic) Edema

Ionic (Osmotic) edema results due to the development of an ionic gradient leading to movement
of water from the extracellular to intracellular spaces. This type of edema is seen in cases of
Syndrome of inappropriate antidiuretic hormone (SIADH) or other conditions leading to
hyponatremia. Vasogenic, Hydrostatic, Interstitial, and Amyloid-related edema all result in an
increase in cerebral extracellular fluid.
Cytotoxic edema, or cellular swelling, manifests minutes after acute central nervous system
(CNS) injuries. Ionic edema, an extracellular edema that occurs in the presence of an intact blood
brain barrier (BBB), forms immediately following cytotoxic edema. Vasogenic edema, an
extracellular edema that includes extravasation of plasma proteins, manifests hours after the
initial insult.
66. When treating a patient with acute symptomatic hyponatremia with hypertonic saline, the
maximum correction of plasma sodium levels in the first 24 hours is

- 1-4 mEq/L
- No current guidelines
- 4-8 mEq/L
- >15 mEq/L
- 12-15 mEq/L
4-8 mEq/L
Discussion:
Sodium correction is based on the acuity of hyponatremia and the severity of symptoms. In acute
symptomatic hyponatremia, a correction of 4-6 mEq/L (up to 8mEq/L) is safe and can be
achieved within a few hours, with maintenance of stable sodium levels for up to 24 hours. A
more rapid correction can lead to osmotic demyelination syndrome with neurologic sequelae
thereof.
References:
Sterns RH, Nigwekar SU, Hix JK. The treatment of hyponatremia. Semin Nephrol. 2009
May;29(3):282-99; Verbalis JG, Goldsmith SR, Greenberg A, Korzelius C, Schrier RW, Sterns
RH, Thompson CJ. Diagnosis, evaluation, and treatment of hyponatremia: expert panel
recommendations. Am J Med. 2013 Oct;126(10 Suppl 1):S1-42.
66. Which of the following conditions is most likely to require that a patient's airway be secured
via emergency cricothyroidotomy because of an inability to perform endotracheal intubation?

-Thoracic empyema
-Septic shock
-Diaphragmatic hernia
-Anaphylaxis
-Flail chest
Anaphylaxis
Discussion:
Management of anaphylaxis involves rapid assessment of a patient's airway with emergent
intubation being performed if stridor or respiratory distress occurs due to upper airway
compromise. With anaphylaxis, significant edema of the tongue and oropharyngeal tissues can
occur. Early presence of upper airway edema can represent rapidly developing airway
compromise. In some cases, emergent cricothyroidotomy may be required if severe upper airway
edema prevents access to the glottic aperture, even while using a video laryngoscope. Unless
intubation occurs early when there is minimal disruption of the upper airway anatomy, intubation
should ideally be performed using an awake fiberoptic approach.
References:
1. Soar J, Pumphrey R, Cant A, Clarke S, Corbett A, Dawson P, Ewan P, Foëx B, Gabbott D,
Griffiths M, Hall J, Harper N, Jewkes F, Maconochie I, Mitchell S, Nasser S, Nolan J, Rylance
G, Sheikh A, Unsworth DJ, Warrell D; Working Group of the Resuscitation Council (UK).
Emergency treatment of anaphylactic reactions--guidelines for healthcare providers.
Resuscitation. 2008 May;77(2):157-69. doi: 10.1016/j.resuscitation.2008.02.001. Epub 2008 Mar
20. PMID: 18358585
2. Brown SG, Mullins RJ, Gold MS. Anaphylaxis: diagnosis and management. Med J Aust. 2006
Sep 4;185(5):283-9. doi: 10.5694/j.1326-5377.2006.tb00619.x. Erratum in: Med J Aust. 2006
Oct 2;185(7):400. Dosage error in article text. PMID: 16948628
3. Simons FE, Ardusso LR, Bilò MB, El-Gamal YM, Ledford DK, Ring J, Sanchez-Borges M,
Senna GE, Sheikh A, Thong BY; World Allergy Organization. World Allergy Organization
anaphylaxis guidelines: summary. J Allergy Clin Immunol. 2011 Mar;127(3):587-93.e1-22. doi:
10.1016/j.jaci.2011.01.038. PMID: 21377030.
67. 43F has a generalized seizure and develops severe HTN (SBP > 180 mmHg) immediately
after undergoing resection of a R frontal lobe oligodendroglioma. She is conscious, but is unable
to visually identify objects. All other neurological functions, including pupillary reflexes and
language functions, are normal. An MR image of the brain is shown. Which of the following is
the most likely diagnosis?

-Progressive multifocal leukencephalopathy (PML)


-Posterior cerebral artery (PCA) infarct
-Central retinal artery occlusion (CRAO)
-Artery of percheron infarct
-Posterior reversible encephalopathy syndrome (PRES)
PRES

Discussion:
This patient most likely has posterior reversible encephalopathy syndrome (PRES). PRES results
in impaired cerebral autoregulation. Systemic hypertension is an important contributor to the
blood-brain barrier breakdown seen in PRES. Other factors likely include endothelial
dysfunction, cytokines, vascular endothelial growth factor (VEGF), an imbalance of female
reproductivbe hormones, enhanced oxidative stress, certain single nucleotide polymorphisms,
micro-RNAs, and autonomic dysregulation. PRES results in white matter rarefaction, endothelial
swelling, fibrinoid vascular necrosis, scattered microinfarcts, gliosis, and hemosiderin
deposition. Imaging findings in PRES include MRI T2/fluid-attenuated inversion recovery
(FLAIR) hyperintensities usually located posteriorly in the parietooccipital and frontal lobes.
These lesions are due to reversible vasogenic edema. While most lesions are reversible within
days to weeks, up to one-third of patients develop heterogenous lesions with ares of restricted
diffusion that are not reversible.Up to one-fourth of patients may develop lobar brain
hemorrhages or convexity subarachnoid hemorrhage. Patients who are hypertensive, have renal
failure, are immunocompromised, or who are receiving chemotherapy are at particular risk. In
addition to elevated blood pressure, patients with PRES often present with confusion or
encephalopathy and up to two-thirds develop generalized tonic-clonic seizures. Visual symptoms
can include blurriness, scotomas, and blackout of vision suggesting cortical blindness. Vision
typically recovers except in the relatively rare cases of patients developing occipital infarcts or
hemorrhages. Patients with a central retinal artery occlusion (CRAO) will often have an afferent
pupillary defect (APD) on examination. A posterior cerebral artery (PCA

68. 25M with a remote history of complete C6 SCI has onset of severe autonomic dysreflexia
after his bladder becomes distended. If untreated, which of the following sequelae of autonomic
dysreflexia is most likely to be life-threatening for this patient?

-Hypertensive crisis
-Piloerection
-Profuse diaphoresis and flushing
-Tachycardia
-Severe headache
Hypertensive crisis
Discussion:
Acute hypertensive crisis secondary to autonomic dysreflexia (AD), and autonomic dysreflexia
itself, in patients with spinal cord injuries, is considered a medical emergency that can lead to
seizures, retinal and intracerebral hemorrhages, pulmonary edema, myocardial infarction, and
death. These complications are directly caused by severe, sustained, uncontrolled hypertension.
The other options are other signs or symptoms of AD, not necessarily life-threatening.
References:
Reference (1)Allen KJ, Leslie SW. Autonomic Dysreflexia. [Updated 2022 Feb 14]. In:
StatPearls [Internet]. Treasure Island (FL): StatPearls Publishing; 2022 Jan-
Pubmed Web link
https://www.ncbi.nlm.nih.gov/books/NBK482434/
Reference (2)
Squair JW, Phillips AA, Harmon M, Krassioukov AV. Emergency management of autonomic
dysreflexia with neurologic complications. CMAJ. 2016 Oct 18;188(15):1100-1103. doi:
10.1503/cmaj.151311. Epub 2016 May 24. PMID: 27221275; PMCID: PMC5056874.
Pubmed Web link
https://www.ncbi.nlm.nih.gov/pmc/articles/PMC5056874/
Reference (3)
Bycroft J, Shergill IS, Chung EA, Arya N, Shah PJ. Autonomic dysreflexia: a medical
emergency. Postgrad Med J. 2005 Apr;81(954):232-5. doi: 10.1136/pgmj.2004.024463. Erratum
in: Postgrad Med J. 2005 Oct;81(960):672. Choong, E A L [corrected to Chung, E A L]. PMID:
15811886; PMCID: PMC1743257.
Pubmed Web link
https://pubmed.ncbi.nlm.nih.gov/15811886/

69. The pressor effect of phenylephrine can be blocked by which of the following medications?

-Diltiazem
-Digoxin
-Metoprolol
-Phentolamine
-Amiodarone
Phentolamine
Discussion:
IV Phenylephrine hydrochloride is a sympathomimetic amine that functions primarily as an
alpha-1 adrenergic agonist. It is used for elevating mean arterial pressure by venous and arterial
vasoconstriction and increasing cardiac preload without direct effect on the heart. Due to
phenylephrine having only alpha-receptor stimulation, it can produce baroreceptor-mediated
reflex bradycardia. Out of the options given, Phentolamine is the only medication that is a direct
adrenergic alpha-receptor antagonist, which would counteract the effects of phenylephrine.
References:
Reference (1)Richards E, Lopez MJ, Maani CV. Phenylephrine. [Updated 2022 May 5]. In:
StatPearls [Internet]. Treasure Island (FL): StatPearls Publishing; 2022 Jan-
Pubmed Web link
https://www.ncbi.nlm.nih.gov/books/NBK534801/
Reference (2)
Phentolamine. National Center for Biotechnology Information. PubChem Compound Database.
Pubmed Web link
https://pubchem.ncbi.nlm.nih.gov/compound/Phentolamine

70. 55M on dabigatran is transferred to the emergency department with an intracerebral


hematoma that requires immediate evacuation. Which of the following is the best strategy for
management of this patient's coagulopathy?

-IV administration of 5mg vitamin K and proceed to the operating room


-Administer four-factor prothrombin complex concentrate (4F-PCC) and IV vitamin K and
proceed to the operating room
-IV administration of 5g idarucizumab once on the way to the operating room
-Proceed to the operating room anticipating major hemorrhage as there is no reversal agent
-Emergent hemodialysis on the way to the operating room
IV administration of 5g idarucizumab once on the way to the operating room

Discussion:
Idarucizumab (praxbind) is a humanized antibody fragment against the thrombin-binding site of
dabigatran. Idarucizumab rapidly, durably, and safely reverses the anticoagulant effect of
dabigatran and is the preferred agent for reversal. 4F-PCC is an option for direct oral
anticoagulant reversal and can be utilized in situations when idarucizumab is not available.
Direct oral anticoagulants directly inhibit specific proteins within the coagulation cascade.
Vitamin K is used for warfarin reversal, but its effect is not immediate. Hemodialysis can be
used to clear dabigatran in patients with renal failure, and can be utilized after idarucizumb
administration to clear dabigatran-idarucizumab complexes in patients with renal failure.
References:
1. Medow JE, Dierks MR, Williams E, Zacko JC. The emergent reversal of coagulopathies
encountered in neurosurgery and neurology: a technical note. Clin Med Res. 2015;13(1):20-31.
doi:10.3121/cmr.2014.1237
2. Frontera JA, Lewin JJ 3rd, Rabinstein AA, Aisiku IP, Alexandrov AW, Cook AM, del Zoppo
GJ, Kumar MA, Peerschke EI, Stiefel MF, Teitelbaum JS, Wartenberg KE, Zerfoss CL.
Guideline for Reversal of Antithrombotics in Intracranial Hemorrhage: A Statement for
Healthcare Professionals from the Neurocritical Care Society and Society of Critical Care
Medicine. Neurocrit Care. 2016 Feb;24(1):6-46. doi: 10.1007/s12028-015-0222-x. PMID:
26714677.
3. Pollack CV Jr, Reilly PA, van Ryn J, Eikelboom JW, Glund S, Bernstein RA, Dubiel R,
Huisman MV, Hylek EM, Kam CW, Kamphuisen PW, Kreuzer J, Levy JH, Royle G, Sellke FW,
Stangier J, Steiner T, Verhamme P, Wang B, Young L, Weitz JI. Idarucizumab for Dabigatran
Reversal - Full Cohort Analysis. N Engl J Med. 2017 Aug 3;377(5):431-441. doi:
10.1056/NEJMoa170727
71. 55M on rivaroxaban is transferred to the emergency department with an intracerebral
hematoma that requires immediate evacuation. Which of the following is the best strategy for
management of this patient's coagulopathy?

-IV administration of 10mg of vitamin K and proceed with surgery


-Transfuse patient with FFP, cryoprecipitate, and platelets and proceed with surgery
-IV administration of idarucizumab and proceed with surgery
-Administration of Andexanet alfa and proceed with surgery
-Proceed with surgery, anticipating major hemorrhage and multiple transfusions given no
reversal agent
Administration of andexanet alfa and proceed with surgery
Discussion:
Patients receiving direct oral anticoagulation (DOAC) with Factor Xa Inhibitors (i.e.
Rivaroxaban or Apixaban) who suffer a life-threatening hemorrhage should emergently receive a
reversal agent with Andexanet-alfa. The DOAC dabigatran is a direct thrombin inhibitor, for
which Idarucizumab is the reversal agent. Vitamin K would help when the patient is
anticoagulated with coumadin, not a DOAC.
References:
Reference (1)ANNEXA-4 Investigators. Andexanet Alfa for Acute Major Bleeding Associated
with Factor Xa Inhibitors. N Engl J Med. 2016 Sep 22;375(12):1131-41.
Pubmed Web link
https://pubmed.ncbi.nlm.nih.gov/27573206/
Reference (2)
American Society of Hematology 2018 guidelines for management of venous thromboembolism:
optimal management of anticoagulation therapy. Blood Adv (2018) 2 (22): 3257–3291
Pubmed Web link
https://pubmed.ncbi.nlm.nih.gov/30482765/

72. 25M is brought to the emergency department. He is bleeding profusely from extensive lower
facial fractures. Pulse is 84/min, blood pressure is 110/65 mmHg, and oxygen saturation is 85%.
Which of the following interventions is the most appropriate initial step in management?

-Transfusion of packed red blood cells


-Application of laryngeal mask airway
-Anterior tibial intraosseous cannulation
-Administration of prothrombin complex concentrate
-Performance of cricothyroidotomy
Performance of cricothyroidotomy
Discussion:
In the setting of trauma, the order of airway, breathing, and circulation should always be
remembered. Mandibular fractures, facial swelling, and bleeding lead to difficulty with jaw
thrust and difficulty maintaining a seal during bag-mask ventilation and call for an early
subglottic airway. The provider should presume that preoxygenation with mask ventilation in
patients with facial trauma may be difficult and that reoxygenation during Rapid Sequence
Intubation (RSI) if the first attempt is unsuccessful, may be difficult or impossible. Distortion of
facial structures may make obtaining a seal with a bag valve mask device (BVM) device
difficult. Patients may poorly tolerate positive pressure ventilation, as disruption of tissues may
result in worsening bleeding and, in cases of associated lower airway trauma, significant
subcutaneous emphysema. Practitioners must proceed with the assumption that structural
collapse of the airway may occur during an RSI. The choice of approach is based on the patient’s
ability to maintain a patent airway and oxygenation status. For a “have no time” scenario
(obstructing and hypoxemic), the primary approach may require the performance of
cricothyroidotomy facilitated by a dissociative ketamine dosing. Alternatively, a “double set-up”
may be used: Rapid Sequence Intubation with a single attempt at oral intubation followed
immediately by front of neck airway rescue.
References:
Kovacs G, Sowers N. Airway Management in Trauma. Emerg Med Clin North Am. 2018
Feb;36(1):61-84.
Langeron O, Birenbaum A, Amour J. Airway management in trauma. Minerva Anestesiol. 2009
May;75(5):307-11. PMID: 19412149.
72. A 25M is brought to the ED after sustaining multiple severe injuries in a motor vehicle
collision. Which of the following best predicts the need for massive transfusion to prevent lethal
exsanguination?

-SBP < 110 mmHg


-Open femur fracture
-Pulse pressure < 20 mmHg
-Heart rate > 120/min
-GCS < 9
Heart rate > 120/min
Discussion:
The “ABC score” consists of four dichotomous components that are available at the bedside of
the acutely injured patient early in the assessment phase. The presence of anyone component
contributes 1 point to the total score. Scores range from 0 to 4. The parameters include: 1)
penetrating mechanism, 2) emergency department systolic blood pressure less than 90 mmHg, 3)
emergency department heart rate > 120 beats per minute, and 4) positive FAST. A score of 2 or
greater was used to define “predicted to receive MT.” The only one of these four factors the
patient has is a heart rate over 120. Multiple scales use a cut-off of SBP of 90 (not 110).
References:
Cotton BA, Dossett LA, Haut ER, Shafi S, Nunez TC, Au BK, et al. Multicenter validation of a
simplified score to predict massive transfusion in trauma. J Trauma. 2010 Jul;69 Suppl 1:S33-9.
Ogura T, Nakamura Y, Nakano M, Izawa Y, Nakamura M, Fujizuka K, Suzukawa M, Lefor AT.
Predicting the need for massive transfusion in trauma patients: the Traumatic Bleeding Severity
Score. J Trauma Acute Care Surg. 2014 May;76(5):1243-50. doi:
10.1097/TA.0000000000000200. PMID: 24747455.
73. 32M with severe closed head injury develops a sodium level of 123 mEg/L and becomes
more obtunded. Serum osmolality is 275 mOsmol/kg H2O and he is euvolemic. Which of the
following is the most specific treatment for the suspected diagnosis?

-Urea
-Loop diuretics
-Vaptans
-Hypertonic saline
-Demeclocycline
Vaptans

Discussion:
All of the options are treatments for Syndrome of inappropriate antidiuretic hormone secretion
(SIADH), which is the likely diagnosis here, however vaptans are the most specific treatment
options available for SIADH. Vaptans are vasopressin antagonists that inhibit its effects. In
SIADH, vasopressin is secreted inappropriately in a plasma volume independent fashion.
Selective vasopressin V2-receptor antagonists were shown to be effective in raising sodium
levels in two clinical trials (SALT-1 and SALT-2). Loop diuretics such as furosemide are very
effective at increasing free water excretion. However, this mechanism is not specific for SIADH
and in general loop diuretics are helpful in the acute management of SIADH but are not good
options for long term treatment. Hypertonic saline is invaluable in the treatment of hyponatremia
occurring due to SIADH, especially in patients who are not able to receive loop diuretics (e.g.
aneurysmal subarachnoid hemorrhage patients). Urea normally accounts for about half of the
daily osmolytes excreted. Treatment with urea is able to increase solute excretion and decrease
urine osmolality, however its bitter taste limits clinical utility. Demeclocycline is able to treat
SIADH by inducing nephrogenic diabetes insipidus. Although its precise mechanism of action
remains unclear, it causes nephrogenic diabetes insipidus in about 70% of cases.
Demeclocycline's clinical use is often limited by side effects such as nausea and skin
photosensitivity.
References:
1.Zietse R, van der Lubbe N, Hoorn EJ. Current and future treatment options in SIADH. NDT
Plus. 2009 Nov;2(Suppl_3):iii12-iii19. doi: 10.1093/ndtplus/sfp154. PMID: 19881932; PMCID:
PMC2762827.
2. Marik PE, Rivera R. Therapeutic effect of conivaptan bolus dosing in hyponatremic
neurosurgical patients. Pharmacotherapy
75. One day after undergoing anterior cervical decompression and fusion as treatment for a
severe cervical injury, a 45-year-old man develops acute upper gastrointestinal bleeding. There is
no history of smoking or alcoholism. Which of the following is the most likely cause of this
patient's condition?

-Stress-related mucosal damage


-Acute colonic pseudo-obstruction
-Spontaneous bowel perforation secondary to post-operative ileus
-Spontaneous esophageal rupture
-Esophageal perforation during surgery
Stress-related mucosal damage
Discussion:
Although uncommon, the most likely cause of acute upper gastrointestinal (GI) bleeding in the
post-operative setting is gastritis secondary to stress-related mucosal damage in the upper GI
tract, causing bleeding of gastric and/or duodenal ulcers within the first 48 hours post-op. Other
causes can include exacerbation of established peptic ulcer disease or, less likely, vascular-
enteric communications resulting from inflammatory or infectious processes, which usually
present > 48hrs post-op. Acute colonic pseudo-obstruction and spontaneous bowel perforation
secondary to post-operative ileus would be extremely rare causes of GI bleeding in the first 24hrs
after surgery and would not cause upper GI bleeding.
References:
Reference (1)Jones S, May AK. Postoperative gastrointestinal hemorrhage. Surg Clin North
Am. 2012 Apr;92(2):235-42, viii. doi: 10.1016/j.suc.2012.01.002. Epub 2012 Feb 1. PMID:
22414410.
Pubmed Web link
https://pubmed.ncbi.nlm.nih.gov/22414410/
Reference (2)
Pollard TR, Schwesinger WH, Page CP, Schauer PR, Sirinek KR. Upper gastrointestinal
bleeding following major surgical procedures: prevalence, etiology, and outcome. J Surg Res.
1996 Jul 15;64(1):75-8. doi: 10.1006/jsre.1996.0309. PMID: 8806477.
Pubmed Web link
https://pubmed.ncbi.nlm.nih.gov/8806477/
Reference (3)
Matsumura JS, Prystowsky JB, Bresticker MA, Meyer PR Jr, Joehl RJ, Nahrwold DL.
Gastrointestinal tract complications after acute spine injury. Arch Surg. 1995 Jul;130(7):751-3.
doi: 10.1001/archsurg.1995.01430070073014. PMID: 7611864.
Pubmed Web link
https://pubmed.ncbi.nlm.nih.gov/7611864/
76. 55M on clopidogrel is transferred to the ED with an intracerebral hematoma that requires
immediate evacuation. Which of the following is the best strategy for management of this
patient's coagulopathy?

-IV administration of idarucizumab and proceed with surgery


-IV administration 0.4ug/kg of desmopressin followed by platelet transfusion
-IV administration of 10mg of Vitamin K and proceed with surgery
-Proceed with surgery, anticipating major hemorrhage and multiple transfusions given no
reversal agent
-IV administration of protamine sulfate and proceed with surgery
IV administration 0.4ug/kg of desmopressin followed by platelet transfusion
Discussion:
Although, currently, there is no specific reversal agent for clopidogrel, it is recommended that
patients receive a single dose of 0.4μg/kg of desmopressin intravenously followed by platelet
transfusion. Desmopressin acts by increasing plasma von Willebrand factor, factor VIII and
intracellular platelet calcium/sodium ion concentrations, as well as, increasing formation of
procoagulant platelets and platelet adhesion to collagen. Idarucizumab is the reversal agent for
the direct thrombin inhibitor dabigatran. Protamine sulfate is the reversal agent for heparin.
Vitamin K would help when the patient is anticoagulated with coumadin.
References:
Reference (1)Guideline for Reversal of Antithrombotics in Intracranial Hemorrhage: Executive
Summary. A Statement for Healthcare Professionals From the Neurocritical Care Society and the
Society of Critical Care Medicine. Crit Care Med. 2016 Dec;44(12):2251-2257.
Pubmed Web link
https://pubmed.ncbi.nlm.nih.gov/27858808/
Reference (2)
Desmopressin for treatment of platelet dysfunction and reversal of antiplatelet agents: a
systematic review and meta-analysis of randomized controlled trials. J Thromb Haemost. 2017
Feb;15(2):263-272
Pubmed Web link
https://pubmed.ncbi.nlm.nih.gov/27893176/
77. Which of the following commonly used qualitative tests is most sensitive in detecting the
therapeutic effect of dabigatran?

-Activated clotting time (ACT)


-Thrombin time (TT)
-Activated partial thromboplastin time (aPTT)
-Prothrombin time (PT)
-Bleeding time
Activated partial thromboplastin time (aPTT)
Discussion:
Hemoclot Thrombin Inhibitor (HTI) is the most sensitive test to detect the therapeutic effect of
dabigatran, but this test is not widely available and not commonly used in the clinical setting. Of
the commonly used coagulation tests, activated partial thromboplastin time (aPTT) is highly
sensitive and should be used to monitor dabigatran activity.
References:
Impact of dabigatran on a large panel of routine or specific coagulation assays. Laboratory
recommendations for monitoring of dabigatran etexilate. Thromb Haemost. 2012
May;107(5):985-97
Pubmed Web link
https://pubmed.ncbi.nlm.nih.gov/22438031/
Reference (2)
Roslyn Bonar, Emmanuel J. Favaloro, Soma Mohammed, leonardo pasalic, John Sioufi,
Katherine Marsden. The effect of dabigatran on haemostasis tests: a comprehensive assessment
using in vitro and ex vivo samples. Pathology,Volume 47, Issue 4, 2015, Pages 355-364,ISSN
0031-3025, https://doi.org/10.1097/PAT.0000000000000252.
77. Which of the following molecular signals induces the development of the floor plate of the
spinal cord and specifies the identity of motor neurons?

-Notch
-Retinoic Acid
-Sonic Hedgehog
-Fibroblast Growth Factor
-Hippo
Sonic Hedgehog
Discussion:
The external ectoderm generates the epidermis, whereas the neural crest cells form the peripheral
ganglion, the pigments of the skin, and the dorsal root ganglia. Finally, the neural tube gives rise
to the Central Nervous System, composed of the brain and the spinal cord. Motor neurons are
generated from a specific domain of ventral neural progenitors of the spinal cord. Progenitor
domains are defined by graded Sonic Hedgehog (SHH) signaling regulating the combinatorial
expression of transcription factors. SHH is expressed by the notochord and floor plate. Hippo
signaling is an evolutionarily conserved pathway that controls organ size by regulating cell
proliferation, apoptosis, and stem cell self-renewal. The Notch pathway mediates juxtacrine
cellular signaling wherein both the signal sending and receiving cells are affected through
ligand-receptor crosstalk, by which an array of cell fate decisions in neuronal, cardiac, immune,
and endocrine development are regulated.
References:
Price SR, Briscoe J. The generation and diversification of spinal motor neurons: signals and
responses. Mech Dev. 2004 Sep;121(9):1103-15. doi: 10.1016/j.mod.2004.04.019. PMID:
15296975.
Blum JA, Klemm S, Shadrach JL, Guttenplan KA, Nakayama L, Kathiria A, Hoang analysis of
the adult mouse spinal cord reveals molecular diversity of autonomic and skeletal motor neurons.
Nat Neurosci. 2021 Apr;24(4):572-583. doi: 10.1038/s41593-020-00795-0. Epub 2021 Feb 15.
PMID: 33589834; PMCID: PMC8016743.
78. The MR image shown is obtained from a 32-year-old man with headache, vertigo, and ataxia.
Optimal management includes which of the following?

-Screening for retinal angiomas


-Screening for abdominal masses
-All of the other answers
-Screening for pheochromocytomas
-Genetic analysis
All of the other answers
Discussion:
Hemangioblastomas are uncommon vascular tumors of the CNS. They account for less than 3%
of all CNS tumors and are generally benign, well-circumscribed, but highly vascular, neoplasms.
These lesions appear as a low signal on T1-weighted images and as a high signal on T2-weighted
sequences. Screening for retinal angiomas, abdominal masses, and pheochromocytomas as well
as genetic analysis is recommended for every patient with a newly diagnosed hemangioblastoma.
Follow-up is by MRI of the clinical neuronal region at 6 and 12-24 months postoperatively.
References:
Bamps S, van Calenbergh F, de Vieeschouwer S, et al. What the neurosurgeon should know
about hemangioblastoma, both sporadic and in Von Hippel-Lindau disease: a literature review.
Surg Neurol Int. 2013; 4:145.
Blansfield JA, Choyke L, Morita SY, Choyke PL, Pingpank JF, Alexander HR, et al. Clinical,
genetic and radiographic analysis of 108 patient with von Hippel-Lindau disease manifested by
pancreatic neuroendocrine neoplasms. Surgery. 2007;142:814–8.
78. Timed vital capacity measurements are most definitive in the detection of which of the
following?

-Need for a tracheostomy


-Strength of diaphragm
-Obstructive Lung Disease
-Readiness to wean from a ventilator
-Restrictive lung disease
Obstructive Lung Disease
Discussion:
Timed vital capacity is a test of the vital capacity of the lungs expressed with respect to the
volume of air that can be quickly and forcibly breathed out in a certain amount of time. It is a
sensitive test for obstructive lung disease. It does not help diagnose restrictive lung disease as
that makes it difficult to inhale, not exhale. Timed vital capacity cannot be measured in an
intubated patient.
References:
Sobol BJ, Emirgil C. The First Second Timed vital capacity and the course of obstructive lung
diseases. Chest. 1977 Jul;72(1):81-3. doi: 10.1378/chest.72.1.81. PMID: 872660.
Miller A. Screening tests for pulmonary function abnormality. Environ Health Perspect. 1975
Jun;11:243-6. doi: 10.1289/ehp.7511243. PMID: 1175563; PMCID: PMC1475188.
79. 24F is evaluated because of a 6-month history of irregular menses and headache. Visual field
testing shows bitemporal hemianopia. MR images are shown. Serum prolactin level is 80 ng/mL.
Which of the following is the most appropriate treatment for this patient?

-Start cabergoline and repeat pituitary MRI in 6 months.


-Refer patient for stereotactic radiosurgery.
-Refer patient to endocrinology and gynecology for further evaluation.
-Surgical resection of pituitary adenoma.
-Start bromocriptine and repeat pituitary MRI in 12 months
Surgical resection of pituitary adenoma

Discussion:
The patient presents with a history of headaches, irregular menses, which are concerning for
pituitary dysfunction, signs of chiasmal compression given bitemporal hemianopsia, and elevated
serum prolactin, which are signs of pituitary adenoma. MRI findings are compatible with a
pituitary macroadenoma, given that size is ≥ 1cm. Surgical resection is the recommended
treatment for patients with macroadenomas exhibiting visual field deficits secondary to the
macroadenoma compressing the optic nerve or chiasm and in patients with pituitary apoplexy
with visual disturbances. The goal of surgical resection is to provide symptom relief,
preservation of surrounding structures, and prevention of further deterioration of vision and
pituitary function, as well as, attempting to reverse any already present symptoms. Cabergoline
and Bromocriptine are dopaminergic agonists used as primary therapy for patients with
hyperprolactinemia and prolactinomas with no signs of optic nerve or chiasmatic compression.
Stereotactic radiosurgery is usually considered when the initial interventions fail or when there is
recurrence of disease.
References:
Reference (1)Esposito D, Olsson DS, Ragnarsson O, Buchfelder M, Skoglund T, Johannsson G.
Non-functioning pituitary adenomas: indications for pituitary surgery and post-surgical
management. Pituitary. 2019 Aug;22(4):422-434.

Pubmed Web link


https://pubmed.ncbi.nlm.nih.gov/31011999/

Reference (2)
Casanueva FF, Molitch ME, Schlechte JA, Abs R, Bonert V, Bronstein MD, Brue T,
Cappabianca P, Colao A, Fahlbusch R, Fideleff H, Hadani M, Kelly P, Kleinberg D, Laws E,
Marek J, Scanlon M, Sobrinho LG, Wass JA, Giustina A. Guidelines of the Pituitary Society for
the diagnosis and management of prolactinomas. Clin Endocrinol (Oxf). 2006 Aug;65(2):265-73.
80. A healthy 66M has a type II odontoid fracture. MR imaging shows that the insertion of the
transverse ligament is avulsed. Which of the following is the most appropriate management for
this odontoid fracture?

-External mobilization in halo


-External mobilization in collar
-Posterior C1-C2 fixation
-Anterior odontoid screw fixation with a single screw
-Anterior odontoid screw fixation with two screws
Posterior C1-C2 fixation

Discussion:
Type II odontoid fractures are characterized by a fracture line separating the dens from the body
of C2. Obtaining an MRI protocoled to evaluate craniocervical ligaments may be appropriate if
initial imaging is concerning for malalignment, joint space widening, angulation, disc injury or
clinical concern for high energy injuries. Because the transverse ligament is a key structure in
maintaining the structural integrity of the atlantoaxial joint, its damage in combination with the
fracture suggest that this is an unstable injury. The most appropriate answer for an unstable C1-2
injury is for instrumentation and fusion.

Distractor 1 and 2: anterior odontoid fixation is contraindicated when there is evidence of


transverse ligament rupture as it does not sufficiently address instability

Distractor 3 and 4: External immobilization may be an appropriate initial conservative measure


in linear, non-displaced type II odontoid fractures without ligamentous injury, but not in this
case.
References:
Reference (1)1. Ryken TC, Hadley MN, Aarabi B, Dhall SS, Gelb DE, Hurlbert RJ, Rozzelle CJ,
Theodore N, Walters BC. Management of isolated fractures of the axis in adults. Neurosurgery.
2013;72 Suppl 2:132-50.2. Fehlings MG, Arun R, Vaccaro AR, Arnold PM, Chapman JR,
Kopjar B. Predictors of treatment outcomes in geriatric patients with odontoid fractures:
AOSpine North America multi-centre prospective GOF study. Spine. 2013;38(11):881.
Pubmed Web link
https://pubmed.ncbi.nlm.nih.gov/23417186/

Reference (2)
Joaquim, Andrei F, and Alpesh A Patel. Surgical treatment of Type II odontoid fractures:
anterior odontoid screw fixation or posterior cervical instrumented fusion? Neurosurgical focus
vol. 38,4 (2015): E11. doi:10.3171/2015.1.FOCUS14781
Pubmed Web link
https://pubmed.ncbi.nlm.nih.gov/25828487
81. In a patient who underwent surgery one week ago, which of the following substances
provides wound integrity and strength?
-Scar Tissue
-Collagen
-Fibrin
-Neutrophils
-Macrophages
Collagen
Discussion:
Wound healing is a complex biological process that consists of hemostasis, inflammation,
proliferation (4 days to three weeks, the time frame of this patient), and remodeling. The first
hemostasis phase begins immediately after wounding, with vascular constriction and fibrin clot
formation. Once bleeding is controlled, inflammatory cells migrate into the wound (chemotaxis)
and promote the inflammatory phase, characterized by the sequential infiltration of neutrophils,
macrophages, and lymphocytes. A critical function of neutrophils is the clearance of invading
microbes and cellular debris. Macrophages are also responsible for inducing and clearing
apoptotic cells, thus paving the way for the resolution of inflammation. In the proliferative phase,
fibroblasts and endothelial cells are the most prominent cell types present and support capillary
growth, collagen formation, and the formation of granulation tissue at the site of injury. Within
the wound bed, fibroblasts produce collagen and glycosaminoglycans and proteoglycans, major
components of the extracellular matrix (ECM). Scar tissue occurs during the remodeling phase
and occurs over months to years.
References:
Guo S, Dipietro LA. Factors affecting wound healing. J Dent Res. 2010 Mar;89(3):219-29. doi:
10.1177/0022034509359125. Epub 2010 Feb 5. PMID: 20139336; PMCID: PMC2903966.
Frantz C, Stewart KM, Weaver VM. The extracellular matrix at a glance. J Cell Sci. 2010 Dec
15;123(Pt 24):4195-200. doi: 10.1242/jcs.023820. PMID: 21123617; PMCID: PMC2995612.
82. According to the Brain Trauma Foundation Guidelines for the Management of Severe
Traumatic Brain Injury, level I evidence exists for which of the following statements?

-Feeding patients to attain basal caloric replacement by the fifth day is recommended to decrease
mortality
-Prolonged prophylactic hyperventilation is not recommended
-Treating ICP above 22 mmHg is recommended to decrease mortality
-Continuous drainage of CSF through an external ventricular drain may be considered to lower
intracranial pressure (ICP)
-Use of steroids is not recommended for improving outcome or reducing ICP
Use of steroids is not recommended for improving outcome or reducing ICP
Discussion:
Continuous drainage of CSF through an external ventricular drain is a level 3 recommendation in
the updated brain trauma foundation guidelines, whereas feeding patients to attain basal caloric
replacement by the fifth day is level 2a, and prolonged prophylactic hyperventilation and treating
ICP above 22 mmHg are level 2b. The only level I evidence is to avoid the use of steroids in the
treatment of traumatic brain injury in the Brain Trauma Foundation Guidelines for the
Management of Severe Traumatic Brain Injury.
References:
Carney N, Totten AM, O'Reilly C, Ullman JS, Hawryluk GW, Bell MJ, Bratton SL, Chesnut R,
Harris OA, Kissoon N, Rubiano AM, Shutter L, Tasker RC, Vavilala MS, Wilberger J, Wright
DW, Ghajar J. Guidelines for the Management of Severe Traumatic Brain Injury, Fourth Edition.
Neurosurgery. 2017;80(1):6-15
The Brain Trauma Foundation. The American Association of Neurological Surgeons. The Joint
Section on Neurotrauma and Critical Care. Role of steroids. J Neurotrauma. 2000 Jun-Jul;17(6-
7):531-5.
83. 17M who sustained a severe brain injury in a MVA is becoming progressively hypotensive.
A Swan-Ganz catheter shows a cardiac output of 4.0L/min, a pulmonary capillary wedge
pressure of 4.0 mmHg, and a systemic vascular resistance of 1400 dynes/sec/cm5. Which of the
following is the most likely diagnosis?
Hypovolemia

Discussion:
Normal range for SVR is 800-1200 synes-s/cm5. Wedge pressure is normally 6-18 mm Hg.
Cardiac output is normally 4-8 L/min. This young patient with a traumatic brain injury (TBI) is
becoming progressively hypotensive with a cardiac output that is on the lower end or norma, a
low wedge pressure, and an increased SVR. This set of monitoring values is most consistent with
hypovolemia, which commonly occurs after polytrauma and TBI. Physical examination findings
are also likely to demonstrate cool limbs, dry mucus membranes, and flat neck veins. Patients
with distributive shock will also have low wedge pressures but typically have decreased SVR.
Patients with cardiogenic shock have increased wedge pressure as well as increased SVR.
Obstructive shock can be due to a variety of conditions such as pulmonary embolism, acute or
worsening pulmoanry hypertension, constrictive pericarditis, restrictive cardiomyopathy, or
tension pneumothorax. Obstructive shock can mirror cardiogenic shock on clinical examination
with the absence of pulmonary rales. Patients with cardiac tamponade have elevated wedge
pressures and often demonstrate tachycardia initially followed by the development of pulsus
paradoxus and refractory hypotension.
References:
1. Simmons J, Ventetuolo CE. Cardiopulmonary monitoring of shock. Curr Opin Crit Care. 2017
Jun;23(3):223-231. doi: 10.1097/MCC.0000000000000407. PMID: 28398907; PMCID:
PMC5678958.
2. Singer M, Deutschman CS, Seymour CW, Shankar-Hari M, Annane D, Bauer M, Bellomo R,
Bernard GR, Chiche JD, Coopersmith CM, Hotchkiss RS, Levy MM, Marshall JC, Martin GS,
Opal SM, Rubenfeld GD, van der Poll T, Vincent JL, Angus DC. The Third International
Consensus Definitions for Sepsis and Septic Shock (Sepsis-3). JAMA. 2016 Feb 23;315(8):801-
10. doi: 10.1001/jama.2016.0287. PMID: 26903338; P
85. Which of the following reverses the effects of dabigatran?

-Idarucizumab
-Protamine
-High dose of vitamin K and fresh frozen plasma
-Recombinant factor Xa
-Platelet transfusion
Idarucizumab

Discussion:
Idarucizumab (praxbind) is a humanized antibody fragment against dabigatran and adheres to the
thrombin-binding site. It has a much higher affinity for dabigatran than thrombin, and the
idarucizumab-dabigatran complex is then cleared by the kidneys. Idarucizumab has been shown
to rapidly and safely reverse the effects of dabigatran in patients who presented with
uncontrolled bleeding or who were about to undergo an urgent procedure in the RE-VERSE AD
trial. As idarucizumab is the most specific and fastest reversal agent for dabigatran, it is the
preferred option for reversal. Idarucizumab (5 g IV in two divided doses) is recommended for
reversal if dabigatran was administered within a period of 3-5 half-lives and there is no evidence
of renal failure. If idarucizumab is not available, activated PCC (50 U/kg) is recommended.
Idarucizumab can be redosed in patients with ongoing clinically significant bleeding with the
addition of hemodialysis if renal failure is present. FFP can be used for warfarin reversal if
activated PCC is not available. Platelet transfusions are useful in the reversal of antiplatelet
administration. Protamine is used for reversal of unfractionated heparin and low molecular
weight heparin. Recombinant factor Xa (andexxa) can be used for the reversal of factor Xa
inhibitors such as rivaroxaban, apixaban, and edoxaban. Its use is the subject of ongoing clinical
investigation.
References:
1. Frontera JA, Lewin JJ 3rd, Rabinstein AA, Aisiku IP, Alexandrov AW, Cook AM, Del Zoppo
GJ, Kumar M, Peerschke EI, Stiefel MF, Teitelbaum JS, Wartenberg KE, Zerfoss CL. Guideline
for Reversal of Antithrombotics in Intracranial Hemorrhage: Executive Summary. A Statement
for Healthcare Professionals From the Neurocritical Care Society and the Society of Critical
Care Medicine. Crit Care Med.
86. 45F is admitted with a small R anterior temporal lobe contusion after an assault. Which of
the following benefits do prophylactic anticonvulsants provide to this patient?

-The use of prophylactic antiepileptics provide no benefit in this patient population


-Reduce the risk of early post-traumatic seizures
-Reduce the risk of post-traumatic epilepsy
-Reduce the risk of late post-traumatic seizures
-Reduce risk of status epilepticus in the first 24 hours following injury
Reduce the risk of early post-traumatic seizures
Discussion:
The prognosis following an acute symptomatic seizure, in the setting of stroke, traumatic brain
injury (TBI) or Central Nervous System (CNS) infection, differs from that of a first unprovoked
seizure. Patients with acute symptomatic seizures are 9 times more likely to die within 30 days,
when compared to those with a first unprovoked seizure. Post-traumatic seizures are divided in 2
subgroups: early post-traumatic seizure (EPTS), when seizures occur within the first 7 days after
brain injury, and late post-traumatic seizure (LPTS), when seizures occur after the first 7 days of
injury. The use of antiepileptics drugs (AED) has been shown to reduce EPTS, but no benefits
have been demonstrated for its use in LPTS.
References:
Reference (1)Kirmani BF, Robinson DM, Fonkem E, Graf K, Huang JH. Role of
Anticonvulsants in the Management of Posttraumatic Epilepsy. Front Neurol. 2016 Mar 22;7:32.
doi: 10.3389/fneur.2016.00032. PMID: 27047441; PMCID: PMC4801868.
Pubmed Web link
https://pubmed.ncbi.nlm.nih.gov/27047441/
Reference (2)
Torbic H, Forni AA, Anger KE, Degrado JR, Greenwood BC. Use of antiepileptics for seizure
prophylaxis after traumatic brain injury. Am J Health Syst Pharm. 2013 May 1;70(9):759-66.
doi: 10.2146/ajhp120203. PMID: 23592358.
Pubmed Web link
https://pubmed.ncbi.nlm.nih.gov/23592358/
Reference (3)
Hesdorffer DC, Benn EK, Cascino GD, Hauser WA. Is a first acute symptomatic seizure
epilepsy? Mortality and risk for recurrent seizure. Epilepsia. 2009 May;50(5):1102-8. doi:
10.1111/j.1528-1167.2008.01945.x. Epub 2009 Jan 26. PMID: 19374657.
Pubmed Web link
https://pubmed.ncbi.nlm.nih.gov/19374657/
86. In the process of wound healing, which of the following cells arrives at the wound first?

-Fibroblasts
-Neutrophils
-Macrophages
-Platelets
-Lymphocytes
Platelets
Discussion:
The wound healing process has three phases: the inflammatory phase, the proliferative phase,
and the remodeling phase. The inflammatory phase is characterized by hemostasis, chemotaxis,
and increased vascular permeability, limiting further damage, closing the wound, removing
cellular debris and bacteria, and fostering cellular migration. The duration of the inflammatory
stage usually lasts several days. Platelets migrate first, followed by neutrophils, then
macrophages, then lymphocytes. Then, during the proliferative phase (beginning on
approximately day 3), the most important cell is the fibroblast. The proliferative phase is
characterized by the formation of granulation tissue, reepithelialization, and neovascularization.
This phase can last several weeks. The maturation and remodeling phase is where the wound
achieves maximum strength as it matures.
References:
Wallace HA, Basehore BM, Zito PM. Wound Healing Phases. [Updated 2022 Jun 21]. In:
StatPearls [Internet]. Treasure Island (FL): StatPearls Publishing; 2022 Jan-. Available from:
https://www.ncbi.nlm.nih.gov/books/NBK470443/
Bowden LG, Byrne HM, Maini PK, Moulton DE. A morphoelastic model for dermal wound
closure. Biomech Model Mechanobiol. 2016 Jun;15(3):663-81.
Ninan N, Thomas S, Grohens Y. Wound healing in urology. Adv Drug Deliv Rev. 2015 Mar;82-
83:93-105
87. 34F with headache and confusion. Non-contrast (Figure 1) and contrast-enhanced (Figure 2)
CT scans are shown. Neurological examination is stable over the first few hours and vital signs
are within normal ranges. Which of the following is the most appropriate immediate therapeutic
intervention?

-Cerebral angiography
-Aspirin
-Hemicraniectomy
-Heparin infusion
-Steroids
Heparin infusion

Discussion:
The axial nonenhanced CT images shows abnormal hyperattenuation consistent with Cerebral
Venous Thrombosis in the superior sagittal sinus. Axial contrast-enhanced CT image shows the
empty delta sign. The empty delta sign consists of a triangular area of contrast enhancement that
surrounds a hypoattenuating area and represents the thrombus; although this sign classically is
found in the superior sagittal sinus, a similar sign can be seen on sagittal and coronal images of
the transverse sinus. Anticoagulation is the first line, mainstay treatment of cerebral venous
thrombosis, even with the presence of acute hemorrhage. Neither steroids nor aspirin have
anticoagulant properties. Cerebral angiography with or without interventions is not indicated at
this time and might be considered in cases refractory to medical management on a case-by-case
basis. There is no evidence to support hemicraniectomy in this scenario given clinical stabiity
and lack of radiological evidence to warrant surgery.
References:
Connolly SJ, Crowther M, Eikelbloom JW, et al. Full study report of andexanet alfa for bleeding
associated with factor Xa inhibitors. N Engl J Med. 2019;380(14):1326-1335
Christos S, Naples R. Anticoagulation reversal and treatment strategies in major bleeding: update
2016. West J Emerg Med 2016;17(3):264-270
Frontera JA, Lewin JJ 3rd, Rabinstein AA, Aisiku IP, Alexandrov AW, Cook AM, del Zoppo
GJ, Kumar MA, Peerschke EI, Stiefel MF, Teitelbaum JS, Wartenberg KE, Zerfoss CL.
Guideline for Reversal of Antithrombotics in Intracranial Hemorrhage: A Statement for
Healthcare Professionals from the Neurocritical Care Society and Society of Critical Care
Medicine. Neurocrit Care. 2016 Feb;24(1):6-46
Greenberg SM, Ziai WC, Cordonnier C, Dowlatshahi D, Francis B, Goldstein JN, Hemphill JC
3rd, Johnson R
87. Neuronal death in the ischemic penumbra is the most likely to be mediated by which of the
following proteins?

-Nitric oxide
-VEGF
-SDF-1a
-Cytochrome C
-Carbon monoxide
Cytochrome C
Discussion:
Severe oxidative stress includes lipid peroxidation, protein dysfunction, DNA damage and
finally necrosis in the ischemic core, whereas weaker damage in the penumbra elicits
predominately apoptosis. Glutamate, K+, ROS, NO, acidosis, and edema spread the injury and
induce penumbra formation. The relatively long-living ROS (H2O2 and O2–) diffuse between
organelles and cells. They activate both pro-survival and pro-apoptotic signaling cascades. The
oxidative injury of endoplasmic reticulum and mitochondrial membranes stimulates release of
stored Ca2+ and mitochondrial pro-apoptotic proteins such as cytochrome c, Smac/DIABLO,
AIF, endonuclease G into the cytosol (Guo et al., 2011). The cell response to acute injury is
initially performed by proteins present in the cell. However, if the injury is strong and present
proteins are unable to cope with the primary lesion, additional proteins are synthesized. Nitric
oxide and carbon monoxide have been implicated in neuronal regeneration, not apoptosis, and
are not proteins. SDF-1α and VEGF are proteins that mediate neuronal repair and regeneration.
References:
Moon S, Chang MS, Koh SH, Choi YK. Repair Mechanisms of the Neurovascular Unit after
Ischemic Stroke with a Focus on VEGF. Int J Mol Sci. 2021 Aug 9;22(16):8543
Broughton BR, Reutens DC, Sobey CG. Apoptotic mechanisms after cerebral ischemia. Stroke.
2009 May;40(5):e331-9.
Uzdensky AB. Regulation of apoptosis in the ischemic penumbra in the first day post-stroke.
Neural Regen Res. 2020 Feb;15(2):253-254. doi: 10.4103/1673-5374.265546. PMID: 31552891;
PMCID: PMC6905348.
87. A 55M on apixaban is transferred to the ED with an intracerebral hematoma that requires
immediate evacuation. Which of the following is the best strategy for management of this
patient's coagulopathy?

-Emergent hemodialysis on the way to the operating room


-IV administration of 5 mg vitamin K and proceed to the operating room
-Proceed to the operating room anticipating major hemorrhage as there is no reversal agent
-IV administration of 5 g idarucizumab once on the way to the operating room
-IV bolus administration of 800 mg andexanet alfa followed by infusion of 8 mg/min for 120
minutes
IV bolus administration of 800 mg andexanet alfa followed by infusion of 8mg/min for 120
minutes

Discussion:
Apixaban is a direct factor Xa inhibitor. Apixaban is recommended for the prevention of stroke
and systemic embolism in those with non-valvular atrial fibrillation and at least one of the
following risk factors: prior stroke or transient ischemic attack, age 75 years or older, diabetes
mellitus, or symptomatic heart failure. Andexanet alfa is recombinant modified human factor Xa
protein that binds directly to free-floating factor Xa inhibitors with high affinity, sequestering
them, and thus rapidly reducing the free plasma concentration of these agents, neutralizing their
anticoagulant effect. Andexanet alfa is a Food and Drug Administration (FDA) approved
antidote for apixaban in people with uncontrolled and life-threatening bleeding events. Emergent
dialysis and idarucizimab administration are reversal strategies for dabigatran. Vitamin K is part
of the reversal strategy for warfarin along with fresh frozen plasma, or 4 factor prothrombin
complex concentrate in cases in which warfarin needs to be reversed quickly.
References:
Medow JE, Dierks MR, Williams E, Zacko JC. The emergent reversal of coagulopathies
encountered in neurosurgery and neurology: a technical note. Clin Med Res. 2015;13(1):20-31.
doi:10.3121/cmr.2014.1237
Frontera JA, Lewin JJ 3rd, Rabinstein AA, Aisiku IP, Alexandrov AW, Cook AM, del Zoppo
GJ, Kumar MA, Peerschke EI, Stiefel MF, Teitelbaum JS, Wartenberg KE, Zerfoss CL.
Guideline for Reversal of Antithrombotics in Intracranial Hemorrhage: A Statement for
Healthcare Professionals from the Neurocritical Care Society and Society of Critical Care
Medicine. Neurocrit Care. 2016 Feb;24(1):6-46
Greenberg SM, Ziai WC, Cordonnier C, Dowlatshahi D, Francis B, Goldstein JN, Hemphill JC
3rd, Johnson
88. Which of the following is an antioxidant required for collagen synthesis and wound healing?

-Glutathione
-Vitamin E
-N-acetylcysteine
-Vitamin C
-Coenzyme Q10
Vitamin C
Discussion:
Healing of tissues such as bone, tendon, and ligament requires collagen synthesis. Vitamin C
(ascorbic acid) has an important role in connective tissue healing as it is a cofactor for prolyl
hydroxylase and lysyl hydroxylase. These enzymes catalzye the hydroxylation of proline and
lysine amino acid residues within procollagen, subsequently promoting proper folding and
stability of the stable collagen triple helix formation. Vitamin C is also an antioxidant that is able
to neutralize reactive oxygen species.
References:
1. DePhillipo NN, Aman ZS, Kennedy MI, Begley JP, Moatshe G, LaPrade RF. Efficacy of
Vitamin C Supplementation on Collagen Synthesis and Oxidative Stress After Musculoskeletal
Injuries: A Systematic Review. Orthop J Sports Med. 2018 Oct 25;6(10):2325967118804544.
doi: 10.1177/2325967118804544. PMID: 30386805; PMCID: PMC6204628
2. Gref, R., Deloménie, C., Maksimenko, A. et al. Vitamin C–squalene bioconjugate promotes
epidermal thickening and collagen production in human skin. Sci Rep 10, 16883 (2020).
https://doi.org/10.1038/s41598-020-72704-1.
89. Which of the following opioids is most appropriate to enable neurological assessments
because of its rapid metabolism?

-Propofol
-Midazolam
-Pentobarbital
-Remifentanil
-Fentanyl
Remifentanil

Discussion:
Of the given choices, remifentanil has the faster onset of metabolism, followed by propofol,
fentanyl, midazolam and pentoparbital. As a result, it is s most beneficial for a rapid assessement
of neurological examinations. However, its availability and cost may be limited in some
institutions. The main advantage of remifentanil is its extremely brief half-life. The elimination
half-life is 3 to 6 minutes and is independent of dose and duration. Remifentanil is degraded by
nonspecific plasma and tissue esterases and metabolism is not affected by butyrylcholinesterase
deficiency. Propofol is bi-phasic, with its initial half-life being relatively quick, around 40
minutes, and its terminal half-life usually being 4 to 7 hours. Context-sensitive half-time may be
up to 1 to 3 days after a 10-day infusion. Fentanyl's onset of action is less than 60 seconds with a
half-life of 90 minures and duration of action near 30-60 minutes. Its peak effect is 2-5 minutes.
The α-half-life (distribution and redistribution) of midazolam has been recorded as 4 to 18
minutes. The β-half-life (metabolism and excretion) is 1.7 to 2.4 hours. By contrast, diazepam's
β-half-life is 31.3 hours. The half-life of pentobarbital is approximately 20 hours but may be
extended at higher levels.
References:
Fodale V, Schifilliti D, Praticò C, et al. Remifentanil and the brain. Acta Anaesthesiol Scand.
2008 Mar; 52(3):319-26

Opdenakker, O. , Vanstraelen, A. , De Sloovere, V. & Meyfroidt, G. (2019). Sedatives in


neurocritical care: an update on pharmacological agents and modes of sedation. Current Opinion
in Critical Care, 25 (2), 97-104

Karabinis A, Mandragos K, Stergiopoulos S, et al. Safety and efficacy of analgesia-based


sedation with remifentanil versus standard hypnotic-based regimens in intensive care unit
patients
89. For which of the following indications has the United States Food and Drug Administration
approved the use of the serum biomarkers glial fibrillary acidic protein (GFAP) and ubiquitin
carboxy-terminal hydrolase-L1 (UCH-L1)?

-Predicting the outcome from aneurysmal subarachnoid hemorrhage


-Monitoring efficacy of chemotherapy for malignant primary brain tumor
-Evaluation of mild traumatic brain injury
-Evaluation of elevated intracranial pressure
-Monitoring efficacy of treatment for non-convulsive status epilepticus
Evaluation of mild traumatic brain injury

Discussion:
The United States Food and Drug Administration has approved the use of the serum biomarkers
glial fibrillary acidic protein (GFAP) and ubiquitin carboxy-terminal hydrolase-L1 as clinically
validated early time biomarkers for mild traumatic brain injury (mTBI) at early time points. The
elevation of UCH‐L1 and GFAP in biofluids was associated with injury severity and clinical
outcomes. The use of one diagnostic test with these tandem markers was authorized by the FDA
to aid in the diagnosis and care of mTBI patients.

Cellular damage, resulting from brain injury, leads to the release of cell‐type‐specific proteins
into biofluids such as cerebral spinal fluid (CSF), serum, plasma, or blood. Several
characteristics allow biofluid markers to be clinically significant, amongst which is the
availability of the protein and the ability to readily determine and quantify it. Additionally,
biomarkers should increase significantly in the acute phase post‐TBI as compared to control
subjects, should be brain‐specific, and should be highly sensitive, reflecting the severity of the
TBI. Several biomarkers have been identified as indicators of TBI pathophysiological events,
including necrosis (SBDP150, SBDP145, and SNTF), apoptosis (SBDP120), neuronal cell body
injury (UCH‐L1 and NSE), strogliosis/astroglia injury (GFAP), and inflammation (interleukin‐6
and autoantibodies) and neurodegeneration (Tau, pTau).
References:
Samson K. In the Clinic-Traumatic Brain Injury FDA Approves First Blood Test for Brain
Bleeds After Mild TBI/Concussion. Neurology Today. 2018;18(6):1-37.

Wang KKW, Kobeissy FH, Shakkour Z, Tyndall JA. Thorough overview of ubiquitin C-terminal
hydrolase-L1 and glial fibrillary acidic protein as tandem biomarkers recently cleared by US
Food and Drug Admi
See 11 more
1. Which of the following is the best suture material for wound approximation in a patient
with an infected wound?

-Vicryl
-Prolene
-Silk
-Monocryl
-PDS
Prolene
Discussion:
The nearest ideal suture for an infected wound, is a monofilament nonabsorbable suture
(monofilament nylon). It has low infectivity, results in satisfactory wound tissue strength
when used in infected wounds, and retains its strength. Infected, braided sutures of silk,
nylon, and polyglycolic acid, even after 70 days, were seen to contain bacteria and
polymorphonuclear cells when examined electron microscopically. Absorption of silk
and polyglycolic acid and encapsulation of non-absorbable braided nylon was delayed
by the presence of infection. Monofilament nylon, in contrast, was unaffected, a fibrous
capsule having formed by 10 days even in the infected state. Monocryl, Vicryl, and PDS
are all absorbable. Silk and Vicryl are multifilament. Nylon and Prolene and non-
absorbable monofilament.
References:
Bucknall TE, Teare L, Ellis H. The choice of a suture to close abdominal incisions. Eur
Surg Res. 1983;15(2):59-66. doi: 10.1159/000128334. PMID: 6303791.
Dennis C, Sethu S, Nayak S, Mohan L, Morsi YY, Manivasagam G. Suture materials -
Current and emerging trends. J Biomed Mater Res A. 2016 Jun;104(6):1544-59. doi:
10.1002/jbm.a.35683. Epub 2016 Apr 4. PMID: 26860644.
2. Vitamin C is most likely to enhance wound healing in patients in the ICU by promoting
which of the following?

-Deactivation of self-renewal wound healing genes


-Increased fibroblast motility
-Decreased expression of growth factor cytokines
-Increased transcription of pro-inflammatory genes
-Decreased fibroblast proliferation
Increased fibroblast motility
Discussion:
Vitamic C affects wound healing in multiple ways, including activation of self-renewal
genes, decreasing transcription of pro-inflammatory genes, increasing expression of
growth factor cytokines and increasing fibroblast proliferation and motility.
References:
Mohammed BM, Fisher BJ, Kraskauskas D, Ward S, Wayne JS, Brophy DF, Fowler AA
3rd, Yager DR, Natarajan R. Vitamin C promotes wound healing through novel
pleiotropic mechanisms. Int Wound J. 2016 Aug;13(4):572-84
Duarte TL, Cooke MS, Jones GD. Gene expression profiling reveals new protective
roles for vitamin C in human skin cells. Free Radic Biol Med. 2009 Jan 1;46(1):78-87.

3. 73F with paroxysmal atrial fibrillation is receiving rivaroxaban for ischemic stroke
prophylaxis. She sustains an intraparenchymal hemorrhage. Which of the following is
the most appropriate pharmacotherapy for reversing this patient's anticoagulation?

-FFP
-Tranexamic acid
-Idarucizumab
-Vitamin K
-Andexanet alfa
Andexanet alfa

Discussion:
Rivaraoxaban and Apixaban are factor Xa inhibitors used in atrial fibrillation to prevent
thromboembolic events. Andexanet alfa is a recombinant modified human factor Xa
protein that binds directly to free-floating factor Xa inhibitors with high affinity,
sequestering them, and thus reducing the free plasma concentration of these agents
neutralizing their anticoagulant effect. Tranexamic acid is an anti-fibrinlolytic agent with
no mechanism to neutralize the effect of factor Xa inhibitors. Vitamin K and FFP are
synergistically used to replenish and restore hepatically synthesized clotting factors
inhibited by warfarin. Idarucizamab is a humanized monoclonal antibody fragment that
binds to dabigatran with higher affinity, neutralizing its anticoagulant effect.
References:
Connolly SJ, Crowther M, Eikelbloom JW, et al. Full study report of andexanet alfa for
bleeding associated with factor Xa inhibitors. N Engl J Med. 2019;380(14):1326-1335
Christos S, Naples R. Anticoagulation reversal and treatment strategies in major
bleeding: update 2016. West J Emerg Med 2016;17(3):264-270
Frontera JA, Lewin JJ 3rd, Rabinstein AA, Aisiku IP, Alexandrov AW, Cook AM, del
Zoppo GJ, Kumar MA, Peerschke EI, Stiefel MF, Teitelbaum JS, Wartenberg KE,
Zerfoss CL. Guideline for Reversal of Antithrombotics in Intracranial Hemorrhage: A
Statement for Healthcare Professionals from the Neurocritical Care Society and Society
of Critical Care Medicine. Neurocrit Care. 2016 Feb;24(1):6-46
Greenberg SM, Ziai WC, Cordonnier C, Dowlatshahi D, Francis B, Goldstein JN,
Hemphill JC 3rd, Johnson R, Keigher KM, Mack WJ, Mocco J, Newton EJ, Ruff IM,
Sansing LH, Schulman S, Selim MH, Sheth KN, Sprigg N, Sunnerhagen KS; American
Heart Association/American Stroke Association. 2022 Guideline for the Management of
Patients With Spon
4. 55M treated with abciximab is transferred to the emergency department with an
intracerebral hematoma that requires immediate evacuation. Which of the following is
the best strategy for management of this patient's coagulopathy?

-IV administration of 5 mg vitamin K and proceed to the operating room


-Perform platelet transfusion of 2000-2600 mL and proceed to the operating room
-IV bolus administration of 800 mg andexanet alfa followed by infusion of 8mg/min for
120 minutes
-IV administration of 250 mg aminophylline and proceed to the operating room
-IV administration of 20mg methylprednisolone and proceed to the operating room
Perform platelet transfusion of 2000-2600 mL and proceed to the operating room
Discussion:
Abciximab binds to the glycoprotein (GP) IIb/IIIa receptor of human platelets and inhibits
platelet aggregation. Thus, platelet administration would be the most effective strategy
for treating the resulting coagulopathy. Steroids and aminophylline have no role in
coaguloapthy reversal. Vitamin K would not be useful for coagulopathy resulting from
platelet dysfunction.
References:
Medow JE, Dierks MR, Williams E, Zacko JC. The emergent reversal of coagulopathies
encountered in neurosurgery and neurology: a technical note. Clin Med Res.
2015;13(1):20–31. doi:10.3121/cmr.2014.1237
Frontera JA, Lewin JJ 3rd, Rabinstein AA, Aisiku IP, Alexandrov AW, Cook AM, del
Zoppo GJ, Kumar MA, Peerschke EI, Stiefel MF, Teitelbaum JS, Wartenberg KE,
Zerfoss CL. Guideline for Reversal of Antithrombotics in Intracranial Hemorrhage: A
Statement for Healthcare Professionals from the Neurocritical Care Society and Society
of Critical Care Medicine. Neurocrit Care. 2016 Feb;24(1):6-46
5. Which of the following intravenous opioids has the slowest peak onset of effect?

-Sufentanil
-Morphine
-Remifentanil
-Methadone
-Alfentanil
Morphine
Discussion:
Alfentanil and remifentanil are very fast equilibrating opioids with equilibration half-lives
between plasma and effect site of around 1 minute. They are followed by fentanyl and
sufentanil, with equilibration half-lives of about 6 minutes. Methadone equilibrates with a
half-life of about 8 min. Finally, morphine, equilibrates with a half-life of 2–3 h. PK/PD
modeling has advanced the understanding of the time course of the clinical effects of
opioids after various dosing regimens.
References:
Kharasch ED. Intraoperative methadone: rediscovery, reappraisal, and reinvigoration?
Anesth Analg. 2011 Jan;112(1):13-6.2. Lötsch J. Pharmacokinetic–pharmacodynamic
modeling of opioids. J Pain Symptom Manage. 2005:29:S90-103.
Ing Lorenzini K, Daali Y, Dayer P, Desmeules J. Pharmacokinetic-pharmacodynamic
modelling of opioids in healthy human volunteers. a minireview. Basic Clin Pharmacol
Toxicol. 2012 Mar;110(3):219-26. doi: 10.1111/j.1742-7843.2011.00814.x. Epub 2011
Nov 9. PMID: 21995512.

6. 55M on warfarin for atrial fibrillation is transferred to the ED with an intracerebral


hematoma that requires immediate evacuation. Twelve hours prior, his international
normalized ratio (INR) was 4.0. Which of the following is the best strategy for
management of this patient's coagulopathy?

-Administer four-unit fresh frozen plasma (4U-FFP) and proceed to the operating room
-IV administration of 5mg vitamin K and 4F-PCC and proceed to the operating room
-Proceed to the operating room as half-life of warfarin is eight hours and anticoagulation
is already reversed
-Administer 5mg oral vitamin K via gastronomy tube and proceed to the operating room
-Administer four-factor prothrombin complex concentrate (4F-PCC) only and proceed to
the operating room
IV administration of 5mg vitamin K and 4F-PCC and proceed to the operating room
Discussion:
The optimal reversal strategy is to administer 4 factor PCC immediately given its rapid
effect, especially when immediate surgery is required, and concurrent adminstration of
vitamin K to replenish the clotting factors inhibited by warfarin. PCC is unactivated
prothrombin complex concentrate, and 4F PCC is PCC containing coagulation factors II,
VII, IX, X, protein S and protein C. Fresh frozen plasma (FFP) administration is an
alternative but the hemostatic effect is delayed. FFP can take hours to reverse the
anticoagulation effect of warfarin due to the volume required. Vitamin K administration
by itself would not provide immediate hemostasis.
References:
Medow JE, Dierks MR, Williams E, Zacko JC. The emergent reversal of coagulopathies
encountered in neurosurgery and neurology: a technical note. Clin Med Res.
2015;13(1):20–31. doi:10.3121/cmr.2014.1237
Frontera JA, Lewin JJ 3rd, Rabinstein AA, Aisiku IP, Alexandrov AW, Cook AM, del
Zoppo GJ, Kumar MA, Peerschke EI, Stiefel MF, Teitelbaum JS, Wartenberg KE,
Zerfoss CL. Guideline for Reversal of Antithrombotics in Intracranial Hemorrhage: A
Statement for Healthcare Professionals from the Neurocritical Care Society and Society
of Critical Care Medicine. Neurocrit Care. 2016 Feb;24(1):6-46

6. 25M is brought to the ED immediately after sustaining injuries in a MVC. His BP is


70/40 mmHg and GCS is 6. He is endotracheally intubated with bilateral breath sounds.
His blood pressure increases only transiently in response to IV fluid boluses. Which of
the following is the most likely cause of this patient's persistent hypotension?

-Hypovolemic shock
-Obstructive shock
-Distributive shock
-Septic shock
-Cardiogenic shock
Hypovolemic shock

Discussion:
In the setting of polytrauma, the most likely cause of acute shock state with
minimal/transient response to IV fluids is hypovolemic shock secondary to hemorrhage,
causing a decrease in intravascular volume -> decreased stroke volume -> decreased
cardiac output; the administration of IV fluid bolus will transiently increase intravascular
volume which will, in turn, cause an increase in stroke volume followed by an increase
in cardiac output, but given the likely active exsanguination, this response is only
transient. Obstructive shock, which can be caused by tension pneumothorax or cardiac
tamponade, could be a possible cause, but given bilateral breath sounds after intubation
and the transient improvement in blood pressure after IV fluids would be less likely for
obstructive shock to be the cause. Cardiogenic shock, usually caused by arrhythmias,
acute myocardial infarction among others, would also be less likely. Distributive shock,
which includes septic shock, would be very unlikely in this setting.
References:
Reference (1)Kislitsina ON, Rich JD, Wilcox JE, Pham DT, Churyla A, Vorovich EB,
Ghafourian K, Yancy CW. Shock - Classification and Pathophysiological Principles of
Therapeutics. Curr Cardiol Rev. 2019;15(2):102-113. doi:
10.2174/1573403X15666181212125024. PMID: 30543176
Pubmed Web link
https://www.ncbi.nlm.nih.gov/pmc/articles/PMC6520577/

Reference (2)
Fecher A, Stimpson A, Ferrigno L, Pohlman TH. The Pathophysiology and Management
of Hemorrhagic Shock in the Polytrauma Patient. J Clin Med. 2021 Oct 19;10(20):4793.
doi: 10.3390/jcm10204793. PMID: 34682916
Pubmed Web link
https://www.ncbi.nlm.nih.gov/pmc/articles/PMC8541346/

Reference (3)
Taghavi S, Askari R. Hypovolemic Shock. [Updated 2022 May 1]. In: StatPearls
[Internet]. Treasure Island (FL): StatPearls Publishing; 2022 J

7. A patient p/w lumbar radiculopathy requiring elective surgery. He is on apixaban for a


deep venous thrombosis. Which of the following is the most appropriate amount of time
to wait before operating on the patient?

-4 hours after the last dose was given


-12 hours after the last dose was given
-48 hours after the last dose was given
-24 hours after the last dose was given
-6 hours after the last dose was given
48 hours after the last dose was given

Discussion:
This question relies on two key points of understanding: 1) the urgency of the proposed
surgical intervention, and 2) the half-life of apixaban. The question stem tells us that the
patient has a radiculopathy that will require elective surgery. This does not constitute an
emergency that would require immediate reversal of apixaban, such as andexanet alfa.
Instead, this surgery should be completed as soon as is safe after stopping apixaban - a
decision made with special attention to the indication for the blood thinner and with the
assistance of the physician managing it. When deciding how long to wait for surgery
after administration, the literature suggests that 48 hours is the goal.

Per the manufacturer,


"Apixaban displays prolonged absorption. Thus, despite a short clearance half-life of
about 6 hours, the apparent half-life during repeat dosing is about 12 hours, which
allows twice-daily dosing to provide effective anticoagulation, but it also means that
when the drug is stopped for surgery, anticoagulation persists for at least a day."

The PAUSE trial cited below evaluated the perioperative management of Direct Oral
Anticoagulants in patients with atrial fibrillation who were undergoing elective surgery. In
patients with a low risk for bleeding complications, the authors recommend a 24-hour
hold. In patients with a high risk for bleeding complications, of which neurosurgical
procedures typically fall, a hold period of 48 hours is more appropriate.
References:
Reference (1)Eliquis. Manufacturer Product Information. Bristol-Meyers Squibb.
Princeton, NJ.
Pubmed Web link

Reference (2)
Douketis, James D et al. "Perioperative Management of Patients With Atrial Fibrillation
Receiving a Direct Oral Anticoagulant." JAMA internal medicine vol. 179,11 (2019): 146

8. Which of the following reflects the change in energy requirements and nutritional
substrate that would consequently account for >15% of a patient's daily caloric intake in
the first two weeks after a head injury?

-Increased carbohydrate requirement to maintain euglycemia


-Hypometabolism and decreased protein metabolism requiring less protein
supplementation
-Increased fatty acid oxidation requiring increased saturated fatty acid supplementation
-Hypermetabolism and increased nitrogen excretion
Hypermetabolism and increased nitrogen excretion

Discussion:
It has been known for decades that patients with traumatic brain injury (TBI) have a
hypermetabolic state with hypercatabolism of whole-body protein leading to a highly
negative nitrogen balance, similar to patients with severe polytrauma or extensive
burns. The complex neuro-endocrine-immune response to TBI triggers enhanced
gluconeogenesis secondary to skeletal muscle protein hypercatabolism through
activation of the ubiquitin-proteasome system. This exacerbated whole body protein
catabolism for endogenous calories can be attenuated by the provision of high levels of
dietary protein. However, increasing protein administration in order to block nitrogen
wasting after TBI may result in further elevation of protein catabolism so that only 50%
of the nitrogen supply is retained at high levels of protein intake. Current guidelines
recommend administration of 1.5 to 2.0 g/kg/day of protein accompained by at least
50% of energy needs up to 25-30 kcal/kg/day. Short term permissive underfeeding
while supplying greater levels of protein intake is thought to improve the preservation of
fat-free mass and improve synthetic protein rates.
References:
1. Kurtz and Rocha, Nutrition Therapy, Glucose Control, and Brain Metabolism in
Traumatic Brain Injury: A Multimodal Monitoring Approach.Frontiers in Neuroscience,
March 2020, volume 14, article 190
2. van Zanten ARH, Petit L, De Waele J, Kieft H, de Wilde J, van Horssen P, Klebach
M, Hofman Z. Very high intact-protein formula successfully provides protein intake
according to nutritional recommendations in overweight critically ill patients: a double-
blind randomized trial. Crit Care. 2018 Jun 12;22(1):156. doi: 10.1186/s13054-018-
2070-5. PMID: 29895309; PMCID: PMC5998555.
3. McClave SA, Taylor BE, Martindale RG, Warren MM,

9. Dysfunction of which part of the following hypothalamic-pituitary axes is most likely


three to six months after aneurysmal subarachnoid hemorrhage?

-Prolactin
-Adrenal
-Thyroid
-Somatotroph
-Gonadotroph
Somatotroph
Discussion:
Low pituitary gland volume has been shown to occur after SAH and may result from cell
death or diminished cell size. This may contribute to SAH-mediated dysfunction of
hormone-secreting cells in the anteiro pituitary gland. Hypopituitarism may occur in the
acute and subacute phase after SAH in up to 49% of patients with 26% of patients
experiencing chronic issues. Pituitary function, specifically growth-hormone deficiency,
contributes to poor quality of life. Growth-hormone is likely to be the most commonly
affexted axis in SAH patients. Due to the anatomical site with blood supply via the long
hypophysial portal system, the somatotrophs are most vulnerable. Of note,
somatotrophs are the predominant cell type of the anterior pituitary gland and account
for about 45% of pituitary cells.
References:
1. Can A, Gross BA, Smith TR, et al. Pituitary dysfunction after aneurysmal
subarachnoid hemorrhage: a systematic review and meta-analysis. Neurosurgery.
2016;79:253-64.
2. Rass V, Schoenherr E, Ianosi BA, Lindner A, Kofler M, Schiefecker AJ, Lenhart L,
Gaasch M, Pertl MT, Freyschlag CF, Pfausler B, Delazer M, Beer R, Thomé C, Grams
AE, Scherfler C, Helbok R. Subarachnoid Hemorrhage is Followed by Pituitary Gland
Volume Loss: A Volumetric MRI Observational Study. Neurocrit Care. 2020
Apr;32(2):492-501. doi: 10.1007/s12028-019-00764-x. PMID: 31222466; PMCID:
PMC7082384
10. Which of the following pairs are the key factors that determine the total energy
expenditure of spontaneous ventilation?

-Alveolar surface area and diffusion


-Ventilation and perfusion
-Flow and PEEP
-Pulmonary compliance and resistance
-Alveolar-arterial gradient
Pulmonary compliance and resistance

Discussion:
Energy expenditure during breathing is directly related to work of breathing (WOB).
Work is the product of pressure and volume. In respiratory physiology, WOB is typically
associated with inspiraotry effort, as long as expiration remains a passive process.
WOB can be determined by calculating the area under the curve of a pressure-volume
plot. In this case, pressure is the sum of the transpulmonary pressure gradient and the
chest wall pressure gradient. Alveolar surface area and diffusion are critical for gas
exchange but not directly relevant to the work of breathing. The work to move air into
and out of the lungs plus the work to expand the chest wall is the total work of
breathing. Pulmonar compliance is directly related to WOB as it is the willingness of the
lungs to distend described as the change in volume divided by change in pressure.
Airway pressure during inflation is influenced by volume, thoracic compliance, and
resistance to flow. Pressure gradients allow for flow to be generated, but this is not
directly related to calculating WOB. Positive end expiratory pressure (PEEP) can be
intrinsic or extrinsic. PEEP is an expiratory measurement and does not directly affect
WOB. Ventilation and perfusion describe blood flow and airflow to the lungs but are not
directly involved in WOB. The A-a graident measures the differencxe between oxygen
concentration in the alveoli and arterial system. It is not directly related to WOB but is
important in determining the diferential diagnosis for hypoxemia.
References:
1. Magalhães, P., Padilha, G., Moraes, L. et al. Effects of pressure-support ventilation
with different levels of positive end-expiratory in a mild model of acute respiratory
distress syndrome. ICMx 3 (Suppl 1), A573 (2015). https://doi.org/10.1186/2197-425X-
3-S1-
11. Postoperative hypocalcemia is most likely to have which of the
following cardiovascular effects?

-Torsades de pointed (polymorphic ventricular tachycardia)


-Heart failure
-Prolonged QT interval on EKG
-Acute cardiac ischemia
-HoTN
Choose matching definition
Prolonged QT interval on EKG. HoTN may complicate acute hypocalcemia, particularly
when rapidly induced by transfusion of citrated blood or with use of low calcium
dialysate in the patients undergoing renal replacement therapy. Heart failure has been
reported in severe cases but is not the most common occurrence. Hypocalcemia
characteristically causes prolongation of the QT interval on EKG. Hypocalcemia
prolongs phase 2 of the action potential with the impact modulated by the rate of
change of serum calcium concentration and function of the myocyte calcium channels.
Torsades de pointes can be triggered by hypocalcemia but is much less common than
with hypokalemia or hypomagnesemia.
Periodic breathing with phases of hyperpnea alternating with apnea. Cheyne-Stokes
respiration is characterized by phases of hyperpnea alternating with apnea. Respiratory
depth during the hyperpneic phase increases from breath to breath in a crescendo until
a peak is reached followed by a decrescendo. A period of apnea follows which usually
lasts 10-20 seconds. Cheyne-Stokes respiration is seen in metabolic encephalopathies
and with lesions that impair forebrain and diencephalon function. Apnea occurs when
lesions affect the ventral respiratory group in the ventrolateral medulla bilaterally.
Irregular gasping breathing characterizes cluster or ataxic breathing, which is seen with
lesions of the pontomedullary junction. A respiratory pause at full inspiration
characterizes apneusis, which is seen with bilateral pontine lesions. Sustained
hyperventilation is seen with metabolic encephalopathies and rarely in cases of high
brainstem tumors.
Increased fibroblast motility

Discussion:
Vitamic C affects wound healing in multiple ways, including activation of self-renewal
genes, decreasing transcription of pro-inflammatory genes, increasing expression of
growth factor cytokines and increasing fibroblast proliferation and motility.
References:
Mohammed BM, Fisher BJ, Kraskauskas D, Ward S, Wayne JS, Brophy DF, Fowler AA
3rd, Yager DR, Natarajan R. Vitamin C promotes wound healing through novel
pleiotropic mechanisms. Int Wound J. 2016 Aug;13(4):572-84
Duarte TL, Cooke MS, Jones GD. Gene expression profiling reveals new protective
roles for vitamin C in human skin cells. Free Radic Biol Med. 2009 Jan 1;46(1):78-87.
SBP < 100 refractory to IV fluids and vasopressor administration. Corticosteroid use in
septic shock is indicated as adjunctive therapy for hemodynamic support for patients
needing escalating doses of vasopressors after optimal fluid resuscitation. However, it is
not a 1st line treatment of HoTN in septic shock. In adults with septic shock treated with
low dose corticosteroids, short- and longer-term mortality are unaffected, adverse
events increase, but duration of shock, mechanical ventilation and ICU stay are
reduced. The etiology of the septic shock and concurrent use of abx are variables that
have no relevance to the concurrent use of corticosteroids.
Don't know?
1 of 20
Term
12. HIGH YIELD. 63M sustains a hemorrhagic p-fossa stroke and is taken to the OR for
emergency surgical evacuation. the patient is placed in the lateral position for the
procedure. Shortly after elevating the bone flap, the end-tidal CO2 and O2 saturation
decrease, leading to a decrease in the patient's blood pressure. Which of the following
is the most appropriate course of action?
Choose matching definition
Immediately close the wound and prepare for reintubation
Resect cerebellar hemisphere to relieve pressure
Immediate place an EVD at Frazier's point
Flood the field with saline
Increase PEEP
Don't know?
2 of 20
Term
12. 60M is admitted to the ICU after sustaining an inoperative closed
head injury in a MVC. Lab studies show an increased serum
creatinine level, muddy brown casts in urine sediment, and an
increased fractional excretion of sodium. Which of the following is the
most likely diagnosis?

-Dehydration
-Acute tubular necrosis (ATN)
-Acute bladder obstruction
-Kidney laceration
-Cerebral salt wasting
Choose matching definition
ATN. Muddy brown epithelial casts in the urine are pathognomonic for acute tubular
necrosis (ATN). The casts form when either ischemia or toxins cause damage to the
epithelial cells of the renal tubules causing them to slough off and form casts in the
renal tubule. The casts are then eventually excreted into the urine. ATN can be caused
by ischemia (usually due to shock) or toxins.The fractional excretion of sodium is the
percentage of sodium that is excreted into the urine during renal filtration. Intrinsic
kidney failure such as ATN will result in an increased FENa as sodium recovery in the
renal tubule is impaired. Extrinsic causes of renal failure such as dehydration or acute
bladder obstruction will typically have a low FENa, at least in the acute phase. A kidney
laceration would cause blood in the urine. Cerebral salt wasting can be caused by a
closed head injury and may be associated with an elevated FENa. The cause of CSW is
not well understood but has been attributed to perturbations in the sympathetic nervous
system or the release of BNP. It is not, however, a kidney injury in itself and is not
associated with casts in the urine unless there is secondary injury to the kidney due to
severe hypovolemia.
Reduce the risk of early post-traumatic seizures

Discussion:
The prognosis following an acute symptomatic seizure, in the setting of stroke,
traumatic brain injury (TBI) or Central Nervous System (CNS) infection, differs from that
of a first unprovoked seizure. Patients with acute symptomatic seizures are 9 times
more likely to die within 30 days, when compared to those with a first unprovoked
seizure. Post-traumatic seizures are divided in 2 subgroups: early post-traumatic
seizure (EPTS), when seizures occur within the first 7 days after brain injury, and late
post-traumatic seizure (LPTS), when seizures occur after the first 7 days of injury. The
use of antiepileptics drugs (AED) has been shown to reduce EPTS, but no benefits
have been demonstrated for its use in LPTS.
References:
Reference (1)Kirmani BF, Robinson DM, Fonkem E, Graf K, Huang JH. Role of
Anticonvulsants in the Management of Posttraumatic Epilepsy. Front Neurol. 2016 Mar
22;7:32. doi: 10.3389/fneur.2016.00032. PMID: 27047441; PMCID: PMC4801868.
Pubmed Web link
https://pubmed.ncbi.nlm.nih.gov/27047441/

Reference (2)
Torbic H, Forni AA, Anger KE, Degrado JR, Greenwood BC. Use of antiepileptics for
seizure prophylaxis after traumatic brain injury. Am J Health Syst Pharm. 2013 May
1;70(9):759-66. doi: 10.2146/ajhp120203. PMID: 23592358.
Pubmed Web link
https://pubmed.ncbi.nlm.nih.gov/23592358/

Reference (3)
Hesdorffer DC, Benn EK, Cascino GD, Hauser WA. Is a first acute symptomatic seizure
epilepsy? Mortality and risk for recurrent seizure. Epilepsia. 2009 May;50(5):1102-8.
doi: 10.1111/j.1528-1167.2008.01945.x. Epub 2009 Jan 26. PMID: 19374657.
Pubmed Web link
https://pubmed.ncbi.nlm.nih.gov/19374657/

2% NS. This patient is experiencing neurologic deterioration d/t acute symptomatic


hypoNa. In this scenario, Na correction must be promptly, but cautiously, corrected
within a few hours targeting an increase of 4-6 mEq/L (maximum 8 mEq/L ). In acute
symptomatic hypoNa, a correction of 4-6 mEq/L (up to 8 mEq/L) is safe and well
tolerated and can be achieved within a few hours, with close monitoring and
maintenance of levels up to 24h. A more rapid correction can lead to osmotic
demyelination syndrome with neurologic sequelae thereof. A slightly hypertonic saline
solution like 2% normal saline can be administered, since iso-osmotic solutions can
worsen hypoNa by promoting more water retention compared to sodium, potentiating
SIADH. A hypotonic solution like Dextrose 5% can worsen hyponatremia. DDAVP is
useful in diabetes insipidus and thus not in this scenario. A significantly hyperosmolar
solution such as 23% normal saline will over-correct the sodium rapidly, which can be
dangerous.
Distributive Shock - sympathomimetic vasopressors + fluid replacement

Discussion:
This patient most likely is in a state of neurogenic shock, characterized by bradycardia,
hypotension, and low central venous pressures. Neurogenic shock is a state of
imbalance between sympathetic and parasympathetic regulation of cardiac action and
vascular smooth muscle. The dominant signs are profound vasodilation with relative
hypovolemia while blood volume remains unchanged, at least initially. Neurogenic
shock is classified as a type of distributive shock. The primary treatment of
neurogenic/distributive shock is fluid resuscitation (typically balanced crystalloids) and
administration of sympathomimetic (norepinephrine, epinephrine) vasoactive
medications.Hypovolemic shock is a condition of inadequate organ perfusion caused by
loss of intravascular volume, usually acute. Early hypovolemic shock is typically
characterized by tachycardia and hypotension and low central venous
pressures.Cardiogenic shock is primarily a disorder of cardiac function in the form of a
critical reduction of the heart's pumping capacity, caused by systolic or diastolic
dysfunction leading to a reduced ejection fraction or impaired ventricular filling. Patients
with cardiogenic shock may be either tachycardic or bradycardic, however, central
venous pressures are usually high.Obstructive shock is a condition caused by the
obstruction of the great vessels or the heart itself. Although the symptoms resemble
those of cardiogenic shock, obstructive shock needs to be clearly distinguished from the
latter because it is treated quite differently. The treatment of obstructive shock is causal
- thrombolysis of pulmonary embolism, tension pneumothorax or cardiac tamponade by
thoracic/pericardial drainage. Central venous pressures would be expected to be high in
ob
Don't know?
3 of 20
Term
14. A patient p/w lumbar radiculopathy requiring elective surgery. He
is on apixaban for a deep venous thrombosis. Which of the following is
the most appropriate amount of time to wait before operating on the
patient?
Choose matching definition
4 hours after the last dose was given
12 hours after the last dose was given
48 hours after the last dose was given
24 hours after the last dose was given
6 hours after the last dose was given
Don't know?
4 of 20
Term
15. 55M on clopidogrel is transferred to the ED with an intracerebral
hematoma that requires immediate evacuation. Which of the following
is the best strategy for management of this patient's coagulopathy?
Choose matching definition
IV administration of idarucizumab and proceed with surgery
IV administration 0.4ug/kg of desmopressin followed by platelet transfusion
IV administration of 10mg of Vitamin K and proceed with surgery
Proceed with surgery, anticipating major hemorrhage and multiple transfusions given no
reversal agent
IV administration of protamine sulfate and proceed with surgery
Don't know?
5 of 20
Term
16. 20M exhibits a reduction in his ICP when his arterial BP rises. This
is an example of which of the following physiologic phenomena?
Choose matching definition
Hypertensive crisis
Intact cerebral autoregulation
Defective cerebral autoregulation
Cushing's response
Shock
Don't know?
6 of 20
Term
17. 36M has closed fx of femur and small, focal area of SAH after
being involved in MVC. No other cranial injury is noted, and neurologic
exam is normal. After repair of the femur fx, the patient does not
arouse from anesthesia. CTH shows bilateral diffuse, small,
hypodense lesions. Which of the following is most likely cause of the
change in this patient's clinical status?

-Watershed infarcts from hypotension


-Posterior reversible encephalopathy syndrome (PRES)
-Diffuse embolic infarcts
-Diffuse axonal injury (DAI)
-Fat emboli
Choose matching definition
MAP = DBP + 1/3(SBP-DBP).
MAP is average arterial pressure throughout 1 cardiac cycle (systole and diastole). MAP
is influenced by CO and SVR. Most commonly used formula to estimate MAP is: MAP =
DBP + 1/3(SBP-DBP), where SBP = systolic blood pressure and DBP = diastolic blood
pressure.
Fat emboli. The presence of femur fx and diffuse hypodense lesions would be indicative
of fat emboli. One would expect DAI to yield an immediate neurological deficit. There is
no reported HoTN that would yield watershed infarcts, and the close proximity to the
femur repair would make diffuse embolic infarcts and PRES less likely.
Sepsis. There are surprisingly few absolute contra-indications to organ donation, but
sepsis is one of them. Alcoholism and (non-prison) tattoos do not typically exclude
organ donation. Families can choose organ donation for family members who do not
have their donor status explicitly stated on their license or proxy documents. Donation
after cardiac death (DCD) allows for donation in patients who are not brain dead.
Idarucizumab. Many agents have been administered for the reversal of dabigatran, but
the only effective medication is idarucizumab. This is a monoclonal Ab fragment
specifically designed to reverse the anticoagulation effects. The other medications may
have some effect but are not the appropriate therapy in an operative intracranial mass
lesion.
Don't know?
7 of 20
Term
18. When treating a patient with acute symptomatic hyponatremia with
hypertonic saline, the maximum correction of plasma sodium levels in
the first 24 hours is
Choose matching definition
1-4 mEq/L
No current guidelines
4-8 mEq/L
>15 mEq/L
12-15 mEq/L
Don't know?
8 of 20
Term
19. (Bank #2) A 40-year-old woman is being treated with pressure-
support mechanical ventilation following a severe subarachnoid
hemorrhage. Close monitoring of which of the following is most likely
to assure adequate minute ventilation?
Choose matching definition
Inspiratory pressure and positive end-expiratory pressure
Respiratory rate and positive end-expiratory pressure
Tidal Volume
Tidal volume and respiratory rate
Inspiratory pressure and respiratory rate
Don't know?
9 of 20
Term
20. 43F has a generalized seizure and develops severe HTN (SBP >
180 mmHg) immediately after undergoing resection of a R frontal lobe
oligodendroglioma. She is conscious, but is unable to visually identify
objects. All other neurological functions, including pupillary reflexes
and language functions, are normal. An MR image of the brain is
shown. Which of the following is the most likely diagnosis?

-Progressive multifocal leukencephalopathy (PML)


-Posterior cerebral artery (PCA) infarct
-Central retinal artery occlusion (CRAO)
-Artery of percheron infarct
-Posterior reversible encephalopathy syndrome (PRES)
Choose matching definition
IV administration 0.4ug/kg of desmopressin followed by platelet transfusion

Discussion:
Although, currently, there is no specific reversal agent for clopidogrel, it is
recommended that patients receive a single dose of 0.4μg/kg of desmopressin
intravenously followed by platelet transfusion. Desmopressin acts by increasing plasma
von Willebrand factor, factor VIII and intracellular platelet calcium/sodium ion
concentrations, as well as, increasing formation of procoagulant platelets and platelet
adhesion to collagen. Idarucizumab is the reversal agent for the direct thrombin inhibitor
dabigatran. Protamine sulfate is the reversal agent for heparin. Vitamin K would help
when the patient is anticoagulated with coumadin.
References:
Reference (1)Guideline for Reversal of Antithrombotics in Intracranial Hemorrhage:
Executive Summary. A Statement for Healthcare Professionals From the Neurocritical
Care Society and the Society of Critical Care Medicine. Crit Care Med. 2016
Dec;44(12):2251-2257.
Pubmed Web link
https://pubmed.ncbi.nlm.nih.gov/27858808/

Reference (2)
Desmopressin for treatment of platelet dysfunction and reversal of antiplatelet agents: a
systematic review and meta-analysis of randomized controlled trials. J Thromb
Haemost. 2017 Feb;15(2):263-272
Pubmed Web link
https://pubmed.ncbi.nlm.nih.gov/27893176/
PRES

Discussion:
This patient most likely has posterior reversible encephalopathy syndrome (PRES).
PRES results in impaired cerebral autoregulation. Systemic hypertension is an
important contributor to the blood-brain barrier breakdown seen in PRES. Other factors
likely include endothelial dysfunction, cytokines, vascular endothelial growth factor
(VEGF), an imbalance of female reproductivbe hormones, enhanced oxidative stress,
certain single nucleotide polymorphisms, micro-RNAs, and autonomic dysregulation.
PRES results in white matter rarefaction, endothelial swelling, fibrinoid vascular
necrosis, scattered microinfarcts, gliosis, and hemosiderin deposition. Imaging findings
in PRES include MRI T2/fluid-attenuated inversion recovery (FLAIR) hyperintensities
usually located posteriorly in the parietooccipital and frontal lobes. These lesions are
due to reversible vasogenic edema. While most lesions are reversible within days to
weeks, up to one-third of patients develop heterogenous lesions with ares of restricted
diffusion that are not reversible.Up to one-fourth of patients may develop lobar brain
hemorrhages or convexity subarachnoid hemorrhage. Patients who are hypertensive,
have renal failure, are immunocompromised, or who are receiving chemotherapy are at
particular risk. In addition to elevated blood pressure, patients with PRES often present
with confusion or encephalopathy and up to two-thirds develop generalized tonic-clonic
seizures. Visual symptoms can include blurriness, scotomas, and blackout of vision
suggesting cortical blindness. Vision typically recovers except in the relatively rare
cases of patients developing occipital infarcts or hemorrhages. Patients with a central
retinal artery occlusion (CRAO) will often have an afferent pupillary defect (APD) on
examination. A posterior cerebral artery (PCA
Vaptans

Discussion:
All of the options are treatments for Syndrome of inappropriate antidiuretic hormone
secretion (SIADH), which is the likely diagnosis here, however vaptans are the most
specific treatment options available for SIADH. Vaptans are vasopressin antagonists
that inhibit its effects. In SIADH, vasopressin is secreted inappropriately in a plasma
volume independent fashion. Selective vasopressin V2-receptor antagonists were
shown to be effective in raising sodium levels in two clinical trials (SALT-1 and SALT-2).
Loop diuretics such as furosemide are very effective at increasing free water excretion.
However, this mechanism is not specific for SIADH and in general loop diuretics are
helpful in the acute management of SIADH but are not good options for long term
treatment. Hypertonic saline is invaluable in the treatment of hyponatremia occurring
due to SIADH, especially in patients who are not able to receive loop diuretics (e.g.
aneurysmal subarachnoid hemorrhage patients). Urea normally accounts for about half
of the daily osmolytes excreted. Treatment with urea is able to increase solute excretion
and decrease urine osmolality, however its bitter taste limits clinical utility.
Demeclocycline is able to treat SIADH by inducing nephrogenic diabetes insipidus.
Although its precise mechanism of action remains unclear, it causes nephrogenic
diabetes insipidus in about 70% of cases. Demeclocycline's clinical use is often limited
by side effects such as nausea and skin photosensitivity.
References:
1.Zietse R, van der Lubbe N, Hoorn EJ. Current and future treatment options in SIADH.
NDT Plus. 2009 Nov;2(Suppl_3):iii12-iii19. doi: 10.1093/ndtplus/sfp154. PMID:
19881932; PMCID: PMC2762827.
2. Marik PE, Rivera R. Therapeutic effect of conivaptan bolus dosing in hyponatremic
neurosurgical patients. Pharmacotherapy
Evaluation of mild traumatic brain injury

Discussion:
The United States Food and Drug Administration has approved the use of the serum
biomarkers glial fibrillary acidic protein (GFAP) and ubiquitin carboxy-terminal
hydrolase-L1 as clinically validated early time biomarkers for mild traumatic brain injury
(mTBI) at early time points. The elevation of UCH‐L1 and GFAP in biofluids was
associated with injury severity and clinical outcomes. The use of one diagnostic test
with these tandem markers was authorized by the FDA to aid in the diagnosis and care
of mTBI patients.

Cellular damage, resulting from brain injury, leads to the release of cell‐type‐specific
proteins into biofluids such as cerebral spinal fluid (CSF), serum, plasma, or blood.
Several characteristics allow biofluid markers to be clinically significant, amongst which
is the availability of the protein and the ability to readily determine and quantify it.
Additionally, biomarkers should increase significantly in the acute phase post‐TBI as
compared to control subjects, should be brain‐specific, and should be highly sensitive,
reflecting the severity of the TBI. Several biomarkers have been identified as indicators
of TBI pathophysiological events, including necrosis (SBDP150, SBDP145, and SNTF),
apoptosis (SBDP120), neuronal cell body injury (UCH‐L1 and NSE), strogliosis/astroglia
injury (GFAP), and inflammation (interleukin‐6 and autoantibodies) and
neurodegeneration (Tau, pTau).
References:
Samson K. In the Clinic-Traumatic Brain Injury FDA Approves First Blood Test for Brain
Bleeds After Mild TBI/Concussion. Neurology Today. 2018;18(6):1-37.

Wang KKW, Kobeissy FH, Shakkour Z, Tyndall JA. Thorough overview of ubiquitin C-
terminal hydrolase-L1 and glial fibrillary acidic protein as tandem biomarkers recently
cleared by US Food and Drug Admi
Don't know?
10 of 20
Term
15. (Bank #2) Which of the following is the earliest sign of malignant
hyperthermia?
Choose matching definition
Rising end-tidal CO2
Hyperthermia
Metabolic acidosis
Muscle rigidity
Hyperkalemia
Don't know?
11 of 20
Term
20. 23F is admitted to the ICU s/p unrestrained high-speed MVC.
Pulse is 44/min and regular, respirations are 16/min, BP is 68/40
mmHg, and central venous pressure is 2 mmHg. Fluid resuscitation is
initiated, but the patient remains hypotensive and bradycardic. Which
of the following is the most likely cause of shock and the most
appropriate treatment?

-Cardiogenic Shock- sympathomimetic vasopressors + fluid


replacement
-Obstructive Shock- immediate causal treatment
-None of the above
-Distributive Shock- sympathomimetic vasopressors + fluid
replacement
-Hypovolemic Shock- fluid resuscitation (balance crystalloids)
Choose matching definition
Vaptans

Discussion:
All of the options are treatments for Syndrome of inappropriate antidiuretic hormone
secretion (SIADH), which is the likely diagnosis here, however vaptans are the most
specific treatment options available for SIADH. Vaptans are vasopressin antagonists
that inhibit its effects. In SIADH, vasopressin is secreted inappropriately in a plasma
volume independent fashion. Selective vasopressin V2-receptor antagonists were
shown to be effective in raising sodium levels in two clinical trials (SALT-1 and SALT-2).
Loop diuretics such as furosemide are very effective at increasing free water excretion.
However, this mechanism is not specific for SIADH and in general loop diuretics are
helpful in the acute management of SIADH but are not good options for long term
treatment. Hypertonic saline is invaluable in the treatment of hyponatremia occurring
due to SIADH, especially in patients who are not able to receive loop diuretics (e.g.
aneurysmal subarachnoid hemorrhage patients). Urea normally accounts for about half
of the daily osmolytes excreted. Treatment with urea is able to increase solute excretion
and decrease urine osmolality, however its bitter taste limits clinical utility.
Demeclocycline is able to treat SIADH by inducing nephrogenic diabetes insipidus.
Although its precise mechanism of action remains unclear, it causes nephrogenic
diabetes insipidus in about 70% of cases. Demeclocycline's clinical use is often limited
by side effects such as nausea and skin photosensitivity.
References:
1.Zietse R, van der Lubbe N, Hoorn EJ. Current and future treatment options in SIADH.
NDT Plus. 2009 Nov;2(Suppl_3):iii12-iii19. doi: 10.1093/ndtplus/sfp154. PMID:
19881932; PMCID: PMC2762827.
2. Marik PE, Rivera R. Therapeutic effect of conivaptan bolus dosing in hyponatremic
neurosurgical patients. Pharmacotherapy
IV administration 0.4ug/kg of desmopressin followed by platelet transfusion

Discussion:
Although, currently, there is no specific reversal agent for clopidogrel, it is
recommended that patients receive a single dose of 0.4μg/kg of desmopressin
intravenously followed by platelet transfusion. Desmopressin acts by increasing plasma
von Willebrand factor, factor VIII and intracellular platelet calcium/sodium ion
concentrations, as well as, increasing formation of procoagulant platelets and platelet
adhesion to collagen. Idarucizumab is the reversal agent for the direct thrombin inhibitor
dabigatran. Protamine sulfate is the reversal agent for heparin. Vitamin K would help
when the patient is anticoagulated with coumadin.
References:
Reference (1)Guideline for Reversal of Antithrombotics in Intracranial Hemorrhage:
Executive Summary. A Statement for Healthcare Professionals From the Neurocritical
Care Society and the Society of Critical Care Medicine. Crit Care Med. 2016
Dec;44(12):2251-2257.
Pubmed Web link
https://pubmed.ncbi.nlm.nih.gov/27858808/

Reference (2)
Desmopressin for treatment of platelet dysfunction and reversal of antiplatelet agents: a
systematic review and meta-analysis of randomized controlled trials. J Thromb
Haemost. 2017 Feb;15(2):263-272
Pubmed Web link
https://pubmed.ncbi.nlm.nih.gov/27893176/
Distributive Shock - sympathomimetic vasopressors + fluid replacement

Discussion:
This patient most likely is in a state of neurogenic shock, characterized by bradycardia,
hypotension, and low central venous pressures. Neurogenic shock is a state of
imbalance between sympathetic and parasympathetic regulation of cardiac action and
vascular smooth muscle. The dominant signs are profound vasodilation with relative
hypovolemia while blood volume remains unchanged, at least initially. Neurogenic
shock is classified as a type of distributive shock. The primary treatment of
neurogenic/distributive shock is fluid resuscitation (typically balanced crystalloids) and
administration of sympathomimetic (norepinephrine, epinephrine) vasoactive
medications.Hypovolemic shock is a condition of inadequate organ perfusion caused by
loss of intravascular volume, usually acute. Early hypovolemic shock is typically
characterized by tachycardia and hypotension and low central venous
pressures.Cardiogenic shock is primarily a disorder of cardiac function in the form of a
critical reduction of the heart's pumping capacity, caused by systolic or diastolic
dysfunction leading to a reduced ejection fraction or impaired ventricular filling. Patients
with cardiogenic shock may be either tachycardic or bradycardic, however, central
venous pressures are usually high.Obstructive shock is a condition caused by the
obstruction of the great vessels or the heart itself. Although the symptoms resemble
those of cardiogenic shock, obstructive shock needs to be clearly distinguished from the
latter because it is treated quite differently. The treatment of obstructive shock is causal
- thrombolysis of pulmonary embolism, tension pneumothorax or cardiac tamponade by
thoracic/pericardial drainage. Central venous pressures would be expected to be high in
ob
ATN. Muddy brown epithelial casts in the urine are pathognomonic for acute tubular
necrosis (ATN). The casts form when either ischemia or toxins cause damage to the
epithelial cells of the renal tubules causing them to slough off and form casts in the
renal tubule. The casts are then eventually excreted into the urine. ATN can be caused
by ischemia (usually due to shock) or toxins.The fractional excretion of sodium is the
percentage of sodium that is excreted into the urine during renal filtration. Intrinsic
kidney failure such as ATN will result in an increased FENa as sodium recovery in the
renal tubule is impaired. Extrinsic causes of renal failure such as dehydration or acute
bladder obstruction will typically have a low FENa, at least in the acute phase. A kidney
laceration would cause blood in the urine. Cerebral salt wasting can be caused by a
closed head injury and may be associated with an elevated FENa. The cause of CSW is
not well understood but has been attributed to perturbations in the sympathetic nervous
system or the release of BNP. It is not, however, a kidney injury in itself and is not
associated with casts in the urine unless there is secondary injury to the kidney due to
severe hypovolemia.
Don't know?
12 of 20
Term
21. (Bank #1) The American Heart and American Stroke Associations
recommend that intravenous tissue plasminogen activator (tPA) be
administered no later than how long after the onset of acute ischemic
stroke?
Choose matching definition
4.5h
24h
12h
2h
6h
Don't know?
13 of 20
Term
22. 35M comes to ED because of 2d hx of severe H/A. The CT scan
shown is obtained. The patient is awake, alert, and conversing, but
when the patient is returned from the CT scan, he is acutely
unresponsive hypertensive, and has bradycardia. Which of the
following is the most appropriate next step in mgt?
Choose matching definition
Have the patient intubated
Administer atropine
Place an EVD
Repeat head CT
Start IV antihypertensive
Don't know?
14 of 20
Term
22. Which of the following treatments is most likely to decrease the
incidence of chronic subdural hematoma recurrence?
Choose matching definition
Induced hyponatremia
Irrigation of the subdural space
Hyperoxygen therapy
Placement of subdural drain
Bedrest with head of bed flat
Don't know?
15 of 20
Term
23. Inappropriate secretion of ADH can be distinguished from fluid
volume overload most accurately by which of the following?

-Clinical examination
-UNa
-UNa
-SOsm
-UOsm
Choose matching definition
Ventricular arrhythmias.

Discussion:
Mg prevents increases in action potential duration and prolongation of membrane
repolarization. These changes commonly occur after myocardial ischemia and can lead
to ventricular arrhythmias. A-Fib can also be provoked by hypoMg. Magnesium causes
presynaptic inhibition leading to a depressant effect on the CNS. HypoMg results in
renal potassium loss and also suppresses PTH hormone release and activity. HypoMg
therefore often occurs in conjunction w/hypokalemia & hypocalcemia. HypoMg can be
associated w/HTN rather than HoTN.

References:
1. Agus MS, Agus ZS. Cardiovascular actions of magnesium. Crit Care Clin. 2001
Jan;17(1):175-86. 2. Handb Clin Neurol. 2017;141:705-713.
Elevated opening pressure.
Pseudotumor cerebri is a disorder of idiopathic intracranial hypertension (IIH). It often
affects young, obese women who p/w H/A, papilledema, and elevated LP opening
pressure. The chemical and cellular composition of the CSF is usually normal. 1st line
Rx is typically administration of acetazolamide wwo furosemide. If vision is acutely
threatened, temporary CSF drainage by LP or LD may allow a trial of these diuretics.
Options for more permanent treatment are optic nerve sheath (ONS) fenestration and
VPS. Although the latter is the gold standard, it's often fraught w complications over
time in this pt population. The venous sinuses and jugular veins should be imaged as
stenting can be considered for stenosis as an alternative to VPS.
Prolonged QT interval on EKG. HoTN may complicate acute hypocalcemia, particularly
when rapidly induced by transfusion of citrated blood or with use of low calcium
dialysate in the patients undergoing renal replacement therapy. Heart failure has been
reported in severe cases but is not the most common occurrence. Hypocalcemia
characteristically causes prolongation of the QT interval on EKG. Hypocalcemia
prolongs phase 2 of the action potential with the impact modulated by the rate of
change of serum calcium concentration and function of the myocyte calcium channels.
Torsades de pointes can be triggered by hypocalcemia but is much less common than
with hypokalemia or hypomagnesemia.
Clinical examination. Fluid overload can best be distinguished from SIADH by clinical
exam. Dx of SIADH requires an examination c/w euvolemia. Volume overload can be
caused by several conditions such as heart failure, liver failure, nephrotic syndrome, or
renal failure. Clinical features c/w volume overload include jugular venous distention,
peripheral edema, pulmonary edema, and ascites. SOsm will be low in both SIADH &
hypervolemia. UNa is typically >20 mEq/L and urine osmolality > 10 mOsm in SIADH.
While urine sodium is < 20 mEq/L in heart failure, ascites, and nephrotic syndrome,
hypernatremia due to renal failure may present with urine sodium well in excess of 20
mEq/L.
Don't know?
16 of 20
Term
24. 45F is admitted with a small R anterior temporal lobe contusion
after an assault. Which of the following benefits do prophylactic
anticonvulsants provide to this patient?
Choose matching definition
The use of prophylactic antiepileptics provide no benefit in this patient population
Reduce the risk of early post-traumatic seizures
Reduce the risk of post-traumatic epilepsy
Reduce the risk of late post-traumatic seizures
Reduce risk of status epilepticus in the first 24 hours following injury
Don't know?
17 of 20
Term
25. Dysfunction of which part of the following hypothalamic-pituitary
axes is most likely three to six months after aneurysmal subarachnoid
hemorrhage?
Choose matching definition
Prolactin
Adrenal
Thyroid
Somatotroph
Gonadotroph
Don't know?
18 of 20
Term
26. Timed vital capacity measurements are most definitive in the
detection of which of the following?
Choose matching definition
Need for a tracheostomy
Strength of diaphragm
Obstructive Lung Disease
Readiness to wean from a ventilator
Restrictive lung disease
Don't know?
19 of 20
Term
27. (Bank #2) In which of the following types of edema is the
extracellular fluid volume decreased?
Choose matching definition
Ionic (Osmotic) Edema
Amyloid
Vasogenic
Interstitial
Hydrostatic

You might also like